You are on page 1of 137

UNiVErsity of cebu pardo & Talisay

College of Teacher Education


Self- Instructional Module on College and Advance Algebra & Advanced Mathematics

UNIT 1: REAL NUMBER SYSTEM


Unit Overview
We use numbers every day. Sometimes we
notice patterns in numbers, and that’s where
algebra comes in. By using letters to stand for
numbers, we write formulas that help us to predict
properties of real-world objects or processes. This
is a unit on real number system. This unit is
planned to take two weeks. This unit contains
lessons and various activities that correspond with
the given lesson.

Hence, this unit provides a thorough


understanding of the concepts of sets and
properties of real numbers. Lesson 1.1 enables you to understand the concepts of sets and
performed set operations. Lesson 1.2 allows you to classify different sets of real numbers,
performed operations on real numbers and apply the properties of real numbers.

Moreover, at the end of this unit, you should be able to:


 identify the meaning of set symbols;
 use Venn diagram to represent sets, subsets, and set operations;
 perform the different operations of sets;
 graph and write sets in set-builder and interval notation;
 classify numbers as a natural, integer, rational or irrational;
 perform the operations on real numbers; and
 apply the properties of real numbers to specific examples.

2
Prepared by: Rennil S. Bornia MST-Math
Self- Instructional Module on College and Advance Algebra & Advanced Mathematics

Lesson 1.1: Concepts of Sets and Set Operations


I. Desired Learning Outcomes
At the end of the lesson, you will be able to:
• identify the meaning of set symbols;
• explain the correct description of set notations
• use Venn diagrams to represent sets, subsets, and set operations;
• perform the different operations of sets;
• write sets in set-builder notation and interval notation;  graph sets in number
line; and
• solve problem with Venn diagram.

ll. Lesson Overview

Sets is a collection of related objects. You will learn the importance of sets because every
field of mathematics uses or refers to sets in some way. This lesson introduces concepts of
sets, set notations, and union and intersection of sets. You will also learn to use Venn
diagram to show relationships between sets and its uses in solving word problems.

lll. Course Contents and Learning Experiences

A.Activity
Learning Task 1.1.1: Set Notation Matching Game
Instructions: Match the pair of cards above and below.

Source: https://www.transum.org/software/SW/Starter_of_the_day/Students/Set_Notation/

B.Analysis
Learning Task # 1.1.2: Think and Discuss
Instructions: Using the sets A, B, C, and N, determine whether each statement is true or
false. Explain.
𝐴 = {1, 3, 5, 7, 9} 𝐵 = {2, 4, 6, 8} 𝐶 = {1, 2, 3, 4, 5} 𝑁 = {1, 2, 3, … }
1. 3 ∈ 𝐴 2. 3 ∉ 𝐶
3. 𝐶 = 𝑁 4. 𝐴 ≠ 𝐵
3
Prepared by: Rennil S. Bornia MST-Math
Self- Instructional Module on College and Advance Algebra & Advanced Mathematics

5. 𝐵 ⊆ 𝑁 6. 𝐶 ⊈ 𝐴
C. Abstraction

Key Concepts

• Set – is a collection of objects.


 To describe sets with a convenient notation, we use braces { }, and name the
sets with capital letters.
 For example: 𝐴 = {1, 2, 3}
• Elements or members – are the objects or numbers in a set.
 For example: If 𝐴 = {1, 2, 3}, then 1 ∈ 𝐴, 2 ∈ 𝐴, and 3 ∈ 𝐴
• Finite set – a set that has a fixed number of elements
• Infinite set – a set without a fixed number of elements
• Variable – is a letter that is used to stand for some numbers
 For example: the set 𝐵 = {1, 2, 3,…, 49} is written in set-builder notation as B
= {𝑥│𝑥 is a natural number less than 50}
 This notation is read as “B is the set of numbers x such that x is a natural
number less than 50.”  Universal Set – is the set that contains all objects
under consideration.
• Subset – if every element of set A is also a member of set B
 For example: {3, 5} ⊆ {2, 3, 5}, because 3 ∈ {2, 3, 5} and
5 ∈ {2, 3, 5}.

• Union of Sets - is the set of all elements that belong to either A or


B
 In set-builder notation, this is written 𝐴 ∪ 𝐵 = {𝑥|𝑥 ∈ 𝐴
𝑜𝑟 𝑥 ∈ 𝐵}  For example: Let 𝐴 = {2, 3, 4, 5} and 𝐵 =
{0,1,2,3,4}, find 𝐴 ∪ 𝐵. Solution: 𝐴 ∪ 𝐵 = {0,1,2,3,4,5}
(Note than an element that belongs to both sets is listed only once).

• Intersection of Sets – is the set of all elements that are common to


both A and B.  In set-builder notation, this is written 𝐴 ∩ 𝐵 = {𝑥|𝑥
∈ 𝐴 𝑎𝑛𝑑 𝑥 ∈ 𝐵}  For example: Let 𝐴 = {2, 3, 4, 5} and 𝐵 =
{0,1,2,3,4}, find 𝐴 ∩ 𝐵.
Solution: 𝐴 ∩ 𝐵 = {2,3,4}

• Empty set – also known as null set, is the set that contains no elements.
Note: It is not correct to use 0 or {0} as the empty set. The number 0 is not a set and {0}
is a set with one member, the number 0. We use a special symbol ∅ for the empty set.

• Disjoint sets – if the intersection of two sets is the empty set.


4
Prepared by: Rennil S. Bornia MST-Math
Self- Instructional Module on College and Advance Algebra & Advanced Mathematics

 For example: If 𝐴 = {2,3,4} and 𝐵 = {7,8}, then 𝐴 ∩ 𝐵 = ∅ and 𝐴 and


𝐵 are disjoint sets.

• Complement of a Set – denoted by A’, is the set of all elements


in the given universal set that are not in A.
 In a set-builder notation, this is written
.
 For example: If ,
then find A complement or .
Solution: A complement

• Cardinality of a set – is the number of elements in the set.


 For example: If , then find n(A).
Solution: n(A) = 4
• Set builder notation – another method of describing sets.
 In this notation, we use a variable to represent the numbers in the set.
 For example:

 {x│x is an even number}
 is a natural number less than
• Interval Notation – way of describing sets using open or close interval, or half-open
interval. In general, the interval notation

represents all real numbers between and , not including and .This is
an open interval. In {𝑥 |𝑎 < 𝑥 < 𝑏 } set-builder
notation, we write .
The graph of

represents all real numbers between and , including and .This is a closed
interval. In set-builder notation, we write .

The graph of
represents all real numbers between and , not including but including
. This is a half-open interval. In set-builder notation, we write
.

The graph of
represents all real numbers between and , including but not including
. This is a half-open interval. In set-builder notation, we write
.

The graph of

5
Prepared by: Rennil S. Bornia MST-Math
Self- Instructional Module on College and Advance Algebra & Advanced Mathematics

Subsets of the real numbers whose graphs extend forever in one or both directions can
be represented by interval notation using the infinity symbol or the negative
infinity symbol .

represents all real numbers less than .


represents all real numbers greater than .

represents all real numbers less than or equal to .

represents all real numbers greater than or equal to


.
Represents all real numbers.
Example: Graph (−∞, 3]. Write the interval in set-builder notation.
Solution: The set is the real numbers less than or equal to 3. In set-builder notation, this is
the set {𝑥|𝑥 ≤ 3}.
The graph is

• The set {𝑥|𝑥 ≤ −2} ∪ {𝑥|𝑥 > 3} is the set of real numbers that are either less than or equal to −2
or greater than 3. We also could write this in interval notation as (−∞, −2] ∪ (3, ∞). The graph is

• The set {𝑥|𝑥 > −4} ∩ {𝑥|𝑥 < 1} is the set of real numbers that are greater than −4 and less than
1. The interval notation of this set is (−4, 1), which can be written in set-builder notation as {𝑥|
−4 < 𝑥 < 1}. The graph is

Example: Graph the following. Write A and B using interval notation. Write C and D using set-
builder notation.
A. {𝑥|𝑥 ≤ −1} ∪ {𝑥|𝑥 ≥ 2} B. {𝑥|𝑥 ≥ −1} ∩ {𝑥|𝑥 < 5}
C. (−∞, 0) ∪ [1, 3] D. [−1, 3] ∩ (1, 5)
Solution:

A.
(−∞. −1] ∪ [2, ∞)
B.
[−1, 5)

C.
6
Prepared by: Rennil S. Bornia MST-Math
Self- Instructional Module on College and Advance Algebra & Advanced Mathematics

{𝑥|𝑥 < 0} ∪ {𝑥|1 ≤ 𝑥 ≤ 3}


D. The graphs of [−1, 3], in red, and (1, 5), in
blue, are shown on the right
Note that the intersection of the sets occurs where the graphs intersect. Although 1 ∈ [−1, 3],
1 ∉ (1, 5).
Therefore, 1 does not belong to the intersection of the sets. On the other hand, 3 ∈ [−1, 3]
and 3 ∈ (1, 5). Therefore, 3 belongs to the intersection of the sets. Thus we have the
following.

{𝑥|1 < 𝑥 ≤ 3}

D. Applications

Learning Task 1.1.3: Venn Paint


Instructions: Shade the region of the Venn diagram that corresponds to the given set
operations.

Source: https://www.transum.org/Maths/Activity/Venn/

IV. Assessment
Learning Task 1.1.4: Evaluation Quiz
A. Use one of the symbols in each blank to make a true statement.
A = {1, 3, 5, 7, 9} B = {2, 4, 6, 8} C = {1, 2, 3, 4, 5} N = {1, 2, 3, . . .}
1. 6. ____
2.
7. ____
7
Prepared by: Rennil S. Bornia MST-Math
Self- Instructional Module on College and Advance Algebra & Advanced Mathematics

3. 8. 4. 9.
5. 10.
B. Using the sets A, B, C, and D, perform the set operations by listing the elements in each
set.

11.
16.
12.
17.
13.
18.
14. 19.
15. 20.
C. Complete the table below by graphing each set. Also, write sets given in interval
notation in set-builder notation, and write sets given in set-builder notation in interval
notation.

Graph Interval notation Set-builder notation

V. Enhancement

Learning Task 1.1.5: Solving Problems with Venn


Diagrams Instructions:
1. Find a partner.
2. Watch the YouTube video about Solving Problems with Venn Diagrams with the link
https://www.youtube.com/watch?v=MassxXy8iko
3. Survey 20 of your friends regarding their favorite movie genre (horror, action, drama, or
none)
4. They can choose more than one favorite movie genre.
8
Prepared by: Rennil S. Bornia MST-Math
Self- Instructional Module on College and Advance Algebra & Advanced Mathematics

5. Create a Venn diagram to represent the three movie genres and who chooses that
movie genre. (Note: Use only letters to represent the names of your friends)
6. Create another Venn diagram to represent only the number of friends who chooses that
movie genre.
7. Then, answer the following questions?
a.How many of your friends like horror movies?
b.How many of your friends like action movies?
c. How many of your friends chose drama?
d.How many of your friends like both horror and drama?
e.How many of your friends like both action and drama?
f. How many of your friends like both horror and action?
g.How many of your friends chose all the movie genre?
h.How many of your friends like both horror and action but not drama?
i. How many of your friends like both horror and drama but not action?
j. How many of your friends like both action and drama but not horror?

VI. References

Ratti, J. M., & Marcus, S. (2014). College Algebra and Trigonometry (3rd Ed.). Pearson
Education Limited.
Stewart, J., Redlin, L., & Watson, S. (2015). Algebra and trigonometry. Cengage Learning.
https://www.transum.org/software/SW/Starter_of_the_day/Students/Set_Notation/
https://www.transum.org/Maths/Activity/Venn/

Lesson 1.2: The Real Numbers and its Properties

9
Prepared by: Rennil S. Bornia MST-Math
Self- Instructional Module on College and Advance Algebra & Advanced Mathematics

I. Desired Learning Outcomes


At the end of the lesson, you will be able to:
• explain the classifications of each set of real numbers;
• classify numbers as natural, integer, rational or irrational;
• perform operations on real numbers; and
• apply the properties of real numbers to specific examples.

ll. Lesson Overview

The set of real numbers is the basic set of numbers used in algebra. There are many
different types of real numbers. To understand better the set of real numbers, we will study
some of the subsets of numbers that make up this set. This lesson enables you to classify
different sets of real numbers, perform operations on real numbers and understand
applications of real numbers and its properties to real-life example.

lll. Course Contents and Learning Experiences

A.Activity
Learning Task 1.2.1: Look Around!

Fifteen different word


s hidden in this puzzle. How many can you find? Look up, down, across, backward, and
are
diagonally. Figures are scattered around that will serve as c lues to help you locate the mystery words.

0, 1, 2, 3, … … − 3, −2, −1, 0, 1,2 ,3, … 1 4


−1, 0, 1, − , , 0.25, 0.1313, …
2 3

0.25, 0.33 …

2 7
,
3 5
−4, −5, −6, …

0.141414 …,
2.5555 …,
100%,
15%,
25%

̅
𝜋, 𝑒, 3
1, 2, 3, …

10
Prepared by: Rennil S. Bornia MST-Math
Self- Instructional Module on College and Advance Algebra & Advanced Mathematics

Instructions:

11
Prepared by: Rennil S. Bornia MST-Math
Self- Instructional Module on College and Advance Algebra & Advanced Mathematics

B.Analysis
Learning Task # 1.2.2: Think and Discuss
Instructions: True or false? If false, explain
why.
1. All integers are rational.

2. A rational number is a whole number.

3. Some irrational numbers are integers.

4. All irrational numbers are real numbers.

5. No whole numbers are integers.

C. Abstraction

¤ Types of Real Numbers (ℝ)

Real Numbers may be classified as:

Natural Numbers
Counting numbers from one to infinity. {1, 2,3, … }
(ℕ)

Whole Numbers Counting numbers from zero to


{0,1,2,3, … }
(𝕎) infinity.

Positive and Negative numbers


Integers (ℤ) {… − 3, −2, −1,0,1,2,3, … }
(excluding fractions) and zero.

Rational Numbers Integers, fractions, terminating and


(ℚ) repeating decimals.

Irrational Numbers Non-terminating and non-repeating


(ℚ’) decimals.

Note: You can remember the


Operations on Set of Real Numbers
order of operations by the
1. acronym PEMDAS (Please
Perform all operations with grouping symbols first. Grouping symbols
Excuse My Dear Aunt Sally)
include ( ), brackets [ ], braces { }, and fraction Parentheses
bars. Exponents
2. Evaluate exponents or square roots. Multiplication
3. Multiply or divide (from left to right). Division
4. Add or subtract(from left to right
). Addition
Subtraction

12
Prepared by: Rennil S. Bornia MST-Math
Self- Instructional Module on College and Advance Algebra & Advanced Mathematics

Example 1. Find the value of the expression


Solution
8 + 10 2 Given
3( + 4) − 2(5 + √9)
2∙3

= 3 (186 + 4) − 2(52 + √9) Evaluate numerator and denominator

18 Evaluate exponents or
= 3 ( 6 + 4) − 2(25 + 3) square roots

= 3(3 + 4) − 2(25 + 3) Evaluate quotient


= 3(7) − 2(28) Evaluate parentheses
= 21 − 56 Evaluate products
= −35 Evaluate difference

¤ Properties of Real Numbers

Properties For any real numbers 𝒂,𝒃, Examples


and 𝒄
Commutative Property of 𝑎+𝑏=𝑏+𝑎 8+5=5+8
Addition
Commutative Property of
𝑎𝑏 = 𝑏𝑎 4∙6=6∙4
Multiplication
Associative Property of (𝑎 + 𝑏) + 𝑐 = 𝑎 + (𝑏 + 𝑐) (3 + 5) + 8 = 3 + (5 + 8)
Addition
Associative Property of
(𝑎𝑏)𝑐 = 𝑎(𝑏𝑐) (2 ∙ 5)7 = 2(5 ∙ 7)
Multiplication
𝑎(𝑏 + 𝑐) = 𝑎𝑏 + 𝑎𝑐 4(2 + 6) = 4 ∙ 2 + 𝑎 = 4 ∙ 6
Distributive Property
(𝑏 + 𝑐)𝑎 = 𝑎𝑏 + 𝑎𝑐 (5 + 3)9 = 9 ∙ 5 + 9 ∙ 3
Additive Identity Property 𝑎+0=0+𝑎=𝑎 5+0=0+5=5
Multiplicative Identity 𝑎∙1=1∙𝑎=𝑎 12 ∙ 1 = 1 ∙ 12 = 12
Property
Additive Inverse Property 𝑎 + (−𝑎) = −𝑎 + 𝑎 = 0 8 + (−8) = −8 + 8 = 0

Multiplicative Inverse
Property
Multiplication Property of 0∙𝑎=𝑎∙0=0 0∙6=6∙0=0
Zero

Example 2:
a. 2 + (3 + 7) = 2 + (7 + 3) Commutative Property of

13
Prepared by: Rennil S. Bornia MST-Math
Self- Instructional Module on College and Advance Algebra & Advanced Mathematics

Addition
= (2 + 7) + 3 Associative Property of
Addition
b. 2(𝑥 + 3) = 2 ∙ 𝑥 + 2 ∙ 3 Distributive Property
= 2𝑥 + 6 Simplify
c. (𝑎 = 𝑏)(𝑥 + 𝑦) = (𝑎 + 𝑏)𝑥 + (𝑎 + 𝑏)𝑦 Distributive Property
= (𝑎𝑥 + 𝑏𝑥) + (𝑎𝑦 + 𝑏𝑦) Distributive Property
= 𝑎𝑥 + 𝑏𝑥 + 𝑎𝑦 + 𝑏𝑦 Associative Property of
Addition

D. Applications
Learning Task 1.2.3: The Real Me!
Instructions: Construct a diagram showing the relationships amongst the sets of natural
numbers, whole numbers, integers, rational numbers, irrational numbers, and real
numbers. Write three examples for each set in the diagram.

IV. Assessment
Learning Task 1.2.4: Evaluation Quiz
Instructions:
A.Classify each number by checking the box to which each number belongs.
Number Whole Integer Rational Irrational
Number
1. −86
2. 34.74

5. 𝑒

6. −45.37

8. √24

10. 3.2121121112 …

B.Evaluate the arithmetic expression. Show your solution.

14
Prepared by: Rennil S. Bornia MST-Math
Self- Instructional Module on College and Advance Algebra & Advanced Mathematics

14. 1 − 2[3 − 4(√25 − 6 ∙


7)]

C.Determine the property of real numbers being used.


15. 7 + 10 = 10 + 7
16. (−7) + 7 = 0

17. (𝑥 + 2𝑦) + 3𝑧 = 𝑥 + (2𝑦 + 3𝑧) 20. (3 + 𝑥) + 0 = 3 + 𝑥

D. Complete each statement using the property named.


21. 5 + 𝑤 = ____________, commutative property of addition
22. 2𝑥 + 2 = ____________, distributive property
23. 8 ∙ (____) = 8, multiplicative identity property
, multiplicative inverse property
25. (2𝑥 + 3) + 5 = _____________, associative property of addition
26. 3 ∙ (6𝑥) = ____________, commutative property of multiplication
V. Enhancement
Learning Task 1.2.5: Discovery-Discussion-Writing
Instructions:
1. Explain why the sum, the difference, and the product of two rational numbers are
rational numbers. Is the product of two irrational numbers necessarily irrational? What
about the sum?
2. rational or irrational? Is rational or irrational? In general, what can you
say about the sum of a rational and an irrational number? What about the product?
3. We have seen that addition and multiplication are both commutative operations.
a. Is subtraction commutative?
b. Is division of nonzero real numbers commutative?
c. Are the actions of putting on your socks and putting on your shoes commutative?
d. Are the actions of putting on your hat and putting on your coat commutative?
e. Are the actions of washing laundry and drying it commutative?
f. Give an example of a pair of actions that is commutative.
g. Give an example of a pair of actions that is not commutative.

V. References

Ratti, J. M., & Marcus, S. (2014). College Algebra and Trigonometry (3rd Ed.). Pearson
Education Limited. Stewart, J., Redlin, L., & Watson, S. (2015). Algebra and trigonometry.
Cengage Learning.

15
Prepared by: Rennil S. Bornia MST-Math
Self- Instructional Module on College and Advance Algebra & Advanced Mathematics

UNIT 2: ALGEBRAIC EXPRESSIONS


Unit Overview
Algebraic expressions play an important role in
mathematics curriculum. It represents the value of an
expression for all of the values a variable can take on. Take
for example, when you own a burger stand in a city and
your goal is to earn more money than it cost you to run your
business. By finding how much you earn and how much it
cost you to run the business can be modeled with algebraic
expressions. This is a unit on algebraic expressions and will
take two weeks. This unit contains lessons and various
activities that correspond with the given lesson.

Hence, this unit provides a summary of the basic terms of algebraic expressions and
fundamental operations on polynomials. Lesson
2.1 gives you opportunities to perform fundamental operations of polynomials which
includes the special products. In, Lesson 2.2 you will learn to use different methods of
factoring polynomials. Lastly, in Lesson 2.3 you will perform the fundamental operations on
algebraic fractions.

Moreover, at the end of this unit, you should be


able to:  identify the type, term, and degree of
polynomials;
 perform the fundamental operations on polynomials;
 find the product of the ff: sum and difference of two terms, square of binomial and
cube of binomial
 factor polynomials using the ff: common monomial factoring, trial and error method,
special factoring formulas, and grouping.
 perform the fundamental operations on algebraic fractions;  simplify a complex
fraction.

16
Prepared by: Rennil S. Bornia MST-Math
Self- Instructional Module on College and Advance Algebra & Advanced Mathematics

Lesson 2.1: Polynomials


I. Desired Learning Outcomes
At the end of the lesson, you will be able to:
• explain the method of expanding and factoring polynomials;
• identify the type, term, and degree of polynomials;
• perform the fundamental operations on polynomials; and
• find the product of the ff: sum and difference of two terms, square of binomial
and cube of binomial

II. Lesson Overview


Polynomial is an essential part of mathematics and algebra. They are used in every field of
mathematics to express numbers as a result of mathematical operations. In this lesson,
you will understand the basic terms used in algebraic expression including polynomials.
Moreover, you will perform operations on polynomials and apply the special product
formulas to a certain polynomial.

III. Course Contents and Learning Experiences


A.Activity
Learning Task 2.1.1: Polynomial Puzzler
Instructions: Complete each puzzle by filling in the empty spaces. In any row, multiply the
two left spaces to equal the right-hand space. In any column, multiply the two top spaces to
equal the bottom space.

17
Prepared by: Rennil S. Bornia MST-Math
Self- Instructional Module on College and Advance Algebra & Advanced Mathematics

Source: National Council of Teachers of Mathematics (2008). http://illuminations.nctm.org


B.Analysis
Learning Task # 2.1.2: Think and Discuss
Instructions:
1. Did you use expanding method in filling in the empty spaces? Explain how you use this
method.
2. Did you use factoring method in solving the puzzle? Explain how you use this method.
3. Did you use your own strategy as you worked on puzzle aside from expanding and
factoring? If yes, explain your strategy.
4. What is the relationship between expanding and factoring mathematically?

C. Abstraction

Key Concepts
• Algebraic expressions – also known as variable expressions, is an expression that
contains numbers, variables, and operations.
 For example:
• Variable - is a letter or symbol used to represent a quantity that is unknown or that can
change, or vary.
18
Prepared by: Rennil S. Bornia MST-Math
Self- Instructional Module on College and Advance Algebra & Advanced Mathematics

• Terms – the addends of a variable expression.


 The terms for the first expression above are 3𝑥2, −4𝑥𝑦, 5𝑥, − 𝑦, 𝑎𝑛𝑑 − 7.
 Note that the sign of a term is the sign that immediately precedes it.
• Variable terms – are terms of the expression with variables.
 are variable terms from the first expression above.
• Constant term – are terms of the expression without variables.
 The term −7 is a constant term from the first expression above.
• Numerical coefficient – is a constant multiplier of the variables in a term.
 The numerical coefficient for the term 3𝑥2 is 3; the numerical coefficient for the
term −4𝑥𝑦 is −4; the numerical coefficient for the term 5𝑥 is 5; and the
numerical coefficient for the term – 𝑦 is − 1. When the numerical coefficient is 1
or − 1 (as in 𝑥 and − 𝑥 ), the 1 is usually not written.
• Like terms – terms with identical variable parts.
• Monomial – is an expression that contains one term.
• Binomial – is a sum of two monomials.
• Trinomial - is a sum of three monomials.
• Polynomial – sum of monomials
 From the given example of algebraic expression above, the first expression listed
is a polynomial and the other two examples are not.
• Degree of a polynomial - is the highest power of the variable that appears in the
polynomial.

POLYNOMIALS
A polynomial in the variable x is an expression of the form
𝑎𝑛 𝑥𝑛 + 𝑎𝑛−1 𝑥𝑛 −1 + ⋯ + 𝑎1 𝑥 + 𝑎0
where𝑎0 , 𝑎1 , … , 𝑎𝑛 are real numbers, and
𝑛 is a nonnegative integer.𝑎𝑛If ≠ 0,
then the polynomial hasdegree n. The monomials 𝑎𝑘 𝑥𝑘 that make up the
polynomialare called thetermsof the polynomial.

Polynomial Type Terms Degree


2
2𝑥 − 3𝑥 + 4 Trinomial 2
𝑥8 + 5𝑥 Binomial 8
𝑥 , 5𝑥 8
9𝑥 Monomial 9𝑥5 1
6 Monomial 6 0

19
Prepared by: Rennil S. Bornia MST-Math
Self- Instructional Module on College and Advance Algebra & Advanced Mathematics

¤ Addition and Subtraction of Polynomials


 In adding and subtracting polynomials, we use the properties of real numbers
that were discussed in Unit 1.
 Use distributive property and combine like terms (terms with the same variables
raised to the same powers)
 For instance, 5𝑥7 + 3𝑥7 = (5 + 3)𝑥7 = 8𝑥7
Note: In subtracting polynomials, if there is a minus sign before the
parentheses, then change the sign of every term in the parentheses to its
opposite sign when we removed the parentheses.
−(𝑏 + 𝑐) = −𝑏 − 𝑐

Example 1: Find the sum: (𝑥3 − 6𝑥2 + 2𝑥 + 4) + (𝑥3 + 5𝑥2 − 7𝑥).


Solution:
(𝑥3 − 6𝑥2 + 2𝑥 + 4) + (𝑥3 + 5𝑥2 − 7𝑥)
= (𝑥3 + 𝑥3) + (−6𝑥2 + 5𝑥2) + (2𝑥 − 7𝑥) + 4 Group like terms
3 2
= 2𝑥 − 𝑥 − 5𝑥 + 4 Combine like terms
Example 2: Find the difference: (𝑥3 − 6𝑥2 + 2𝑥 + 4) − (𝑥3 + 5𝑥2 − 7𝑥).
Solution:
(𝑥3 − 6𝑥2 + 2𝑥 + 4) − (𝑥3 + 5𝑥2 − 7𝑥)
= 𝑥3 − 6𝑥2 + 2𝑥 + 4 − 𝑥3 − 5𝑥2 + 7𝑥 Distributive Property
3 3 2 2
= (𝑥 − 𝑥 ) + (−6𝑥 − 5𝑥 ) + (2𝑥 + 7𝑥) + 4 Group like terms
2
= −11𝑥 + 9𝑥 + 4 Combine like terms

20
Prepared by: Rennil S. Bornia MST-Math
Self- Instructional Module on College and Advance Algebra & Advanced Mathematics

¤ Multiplication of Algebraic Expressions


 Use distributive property repeatedly and Laws of Exponents when multiplying
polynomials or other algebraic expressions.

Example 3: Find the product: (2𝑥 + 3)(𝑥2 − 5𝑥 + 4).


Solution:
(2𝑥 + 3)(𝑥2 − 5𝑥 + 4)
= 2𝑥(𝑥2 − 5𝑥 + 4) + 3(𝑥2 − 5𝑥 + 4) Distributive
Property
= (2𝑥 ∙ 𝑥2 − 2𝑥 ∙ 5𝑥 + 2𝑥 ∙ 4) + (3 ∙ 𝑥2 − 3 ∙ 5𝑥 + 3 ∙ 4) Distributive
Property
= (2𝑥3 − 10𝑥2 + 8𝑥) + (3𝑥2 − 15𝑥 + 12) Laws of
Exponents
= (2𝑥3) + (−10𝑥2 + 3𝑥2) + (8𝑥 − 15𝑥) + (12) Group like terms
= 2𝑥3 − 7𝑥2 − 7𝑥 + 12 Combine like
terms

¤ Special Product Formulas

SPECIAL PRODUCT FORMULAS


If 𝑎 and 𝑏 are any real numbers of algebraic expressions, then
1. (𝑎 + 𝑏)(𝑎 − 𝑏) = 𝑎2 − 𝑏2 Sum and difference of same terms
2. (𝑎 + 𝑏)2 = 𝑎2 + 2𝑎𝑏 + 𝑏2 Square of a sum of two terms
3. (𝑎 − 𝑏)2 = 𝑎2 − 2𝑎𝑏 + 𝑏2 Square of a difference of two terms
4. (𝑎 + 𝑏)3 = 𝑎3 + 3𝑎2𝑏 + 3𝑎𝑏2 + 𝑏3 Cube of a sum of two terms
5. (𝑎 − 𝑏)3 = 𝑎3 − 3𝑎2𝑏 + 3𝑎𝑏2 − 𝑏3 Cube of a difference of two terms

Example 4: Use a Special Product Formula to find each product.


a. (3𝑥 + 5)2
Solution: Substitute 𝑎 = 3𝑥 𝑎𝑛𝑑 𝑏 = 5 in Product
Formula 2 (3𝑥 + 5)2 = (3𝑥)2 + +2(3𝑥)(5) + 52 = 9𝑥2 + 30𝑥
+ 25

b.
Solution: Substitute in Product Formula 1

c. (𝑥2 − 2)3
Solution: Substitute 𝑎 = 𝑥2 𝑎𝑛𝑑 𝑏 = 2 in Product Formula 5
(𝑥2 − 2)3 = (𝑥2)3 − 3(𝑥2)2(2) + 3(𝑥2)(2)2 − 23 = 𝑥6 − 6𝑥4 + 12𝑥2 − 8
21
Prepared by: Rennil S. Bornia MST-Math
Self- Instructional Module on College and Advance Algebra & Advanced Mathematics

d. (𝑥 + 𝑦 − 1)(𝑥 + 𝑦 + 1)
Solution: If we group (𝑥 + 𝑦) together and think of this as one algebraic expression, we
can use Product Formula 1 with 𝑎 = 𝑥 + 𝑦 and 𝑏 = 1.
(𝑥 + 𝑦 − 1)(𝑥 + 𝑦 + 1) = [(𝑥 + 𝑦) − 1][(𝑥 + 𝑦) + 1]

= (𝑥 + 𝑦)2 − 12 Product Formula 1


= 𝑥2 + 2𝑥𝑦 + 𝑦2 − 1 Product Formula 2

D. Applications

Learning Task 2.1.3: Think-Pair-Solve


Instructions: Solve the following problems.
1. Volume of a Box. An open box is constructed from a 6 in. by 10 in. sheet of cardboard
by cutting a square piece from each corner and then folding up the sides, as shown in the
figure. The volume of the box is
𝑉 = 𝑥(6 − 2𝑥)(10 − 2𝑥)
(a)Explain how the expression for 𝑉 is obtained.
(b)Expand the expression for 𝑉. What is the degree of the resulting polynomial?
(c) Find the volume when 𝑥 = 1 and when 𝑥 = 2.

2. Building Envelope. The building code in a certain town requires that a house be at
least 10 ft from the boundaries of the lot. The buildable area (or building envelope) for the
rectangular lot shown in the figure is given by
𝐴 = (𝑥 − 20)(𝑦 − 20)
(a)Explain how the expression for 𝐴 is obtained.
(b)Expand to express 𝐴 as a polynomial in x and y.
(c)A contractor has a choice of purchasing one of two rectangular lots, each having the
same area. One lot measures 100 ft by 400 ft; the other measures 200 ft by 200 ft.
Which lot has the larger building envelope?

22
Prepared by: Rennil S. Bornia MST-Math
Self- Instructional Module on College and Advance Algebra & Advanced Mathematics

IV. Assessment

Learning Task 2.1.4: Evaluation Quiz


Instructions: Follow instructions in each part and show your solutions clearly in a separate
sheet of paper.
A. Complete the following table by stating whether the polynomial is a monomial, binomial,
or trinomial, then list its terms and state its degree.

Polynomial Type Terms Degree


2
1. 𝑥 − 3𝑥 + 7
2. 2𝑥5 + 4𝑥2
3. −8

4.
5. 𝑥 − 𝑥2 + 𝑥3 − 𝑥4

B. Find the sum, difference, or product.


6. (12𝑥 − 7) − (5𝑥 − 12) 9. (4𝑥 − 5)(2𝑥2 + 7𝑥 − 8)

7. 4(𝑥2 − 3𝑥 + 5) + 3(𝑥2 − 2𝑥 + 1) 10. (5𝑦4 − 3𝑦2 + 9) − (6𝑦4 + 11𝑦2 − 10)

8. 3(𝑥 − 1) + 4(𝑥 + 2)

C. Use the special product formulas to perform the indicated operation.


11. (3𝑥 + 5)(3𝑥 − 5) 14. (𝑥 − 3)3

12. (3𝑥2 − 𝑦)2 15. (3 + 2𝑦)3

13. (4𝑥 + 𝑧)2


23
Prepared by: Rennil S. Bornia MST-Math
Self- Instructional Module on College and Advance Algebra & Advanced Mathematics

D. Perform the
16. (4𝑑 − 1)2 − (2𝑑 − 3)2 19. (√ℎ2 + 1 + 1)(√ℎ2 + 1 − 1) indicated
operation or
2
17. (1 − 𝑏) (1 + 𝑏) 2
20. 5𝑟(1 − 2𝑟) 3 operations and
simplify.
18. (2𝑥 + 𝑦 − 3)(2𝑥 + 𝑦 + 3)

V. Enhancement

Learning Task 2.1.5: Visualizing a Formula


Instructions:
Many of the Special Product Formulas that we
learned in this section can be "seen" as geometrical
facts about length, area, and volume.
For example, the figure on the right shows how the
formula for the square of a binomial can be
interpreted as a fact about areas of squares and
rectangles.
In the figure, 𝑎 and 𝑏 represent lengths, 𝑎2, 𝑏2, 𝑎𝑏, and
(𝑎 + 𝑏)2 represent areas.

1. Explain how the figure verifies the


formula
𝑎2 − 𝑏2 = (𝑎 + 𝑏)(𝑎 − 𝑏).

2. Find a figure that verifies the formula (𝑎 − 𝑏)2 = 𝑎2


− 2𝑎𝑏 + 𝑏2.
3. Explain how the figure verifies the formula
(𝑎 + 𝑏)3 = 𝑎3 + 3𝑎2𝑏 + 3𝑎𝑏2 + 𝑏3.
24
Prepared by: Rennil S. Bornia MST-Math
Self- Instructional Module on College and Advance Algebra & Advanced Mathematics

4. Is it possible to draw a geometric figure that verifies the formula for (𝑎 + 𝑏)4? Explain.
5. a) Expand (𝑎 + 𝑏 + 𝑐)2.
b) Make a geometric figure that verifies the formula you found in part (a).

Source: https://www.stewartmath.com/dp_fops_samples/dp1.html

VI. References

Ratti, J. M., & Marcus, S. (2014). College Algebra and Trigonometry (3rd Ed.). Pearson
Education Limited.
Stewart, J., Redlin, L., & Watson, S. (2015). Algebra and trigonometry. Cengage Learning.
https://www.stewartmath.com/dp_fops_samples/dp1.html
National Council of Teachers of Mathematics (2008). http://illuminations.nctm.org

Lesson 2.2: Factoring Polynomials


I. Desired Learning Outcomes

At the end of the lesson, you will be able to:


• factor out common factors;
• factor general quadratic trinomials using trial and error method and box
method;
• use the factorization theorem to determine whether trinomial is factorable
• use special factoring formulas (differences of two squares, perfect square
trinomial, and sum and difference of two cubes);
• factor polynomials by grouping;
• factor polynomials completely; and  create a flowchart on factoring
polynomials.

II. Lesson Overview

Factoring is a basic math concept that reverses multiplication. Factoring is an essential


and useful skill in real life. Common applications include: dividing something into equal
pieces, exchanging money, comparing prices, understanding time and making calculations

25
Prepared by: Rennil S. Bornia MST-Math
Self- Instructional Module on College and Advance Algebra & Advanced Mathematics

during travel. This lesson provides you with different factoring techniques to rewrite
polynomials in simpler form.

III. Course Contents and Learning Experiences

A.Activity
Learning Task 2.2.1: My Factoring Puzzle
Instructions:
Instructions: Carefully cut apart the puzzle squares. Arrange them by matching correct
factors and equations.
Glue the final arrangement to your My Factoring Puzzle page. Show all your work on a
separate piece of paper.

26
Prepared by: Rennil S. Bornia MST-Math
Self- Instructional Module on College and Advance Algebra & Advanced Mathematics

B.Analysis
Learning Task # 2.2.2:
Instructions: Think and Discuss
1.What is your strategy to match the factors and equation?
2.What things did you consider in factoring?

C. Abstraction

• Factoring
 It is the process of finding the factors of a polynomial.  It is
writing a polynomial as a product of polynomials.

 We say that 𝑥 − 2 and 𝑥 + 2 are factors of 𝑥2 − 4.


• Factoring Out Common Factors
Step 1: Find the greatest common factor of the given terms. The greatest common
factor or GCF is the largest factor that all terms have in common.
Step 2: Factor out (or divide out) the greatest common factor from each term.

Example 1. Factor 3𝑥2 − 6𝑥


Solution: The GCF of the terms 3𝑥2 and 6𝑥 is 3𝑥, so we have
3𝑥2 − 6𝑥 = 3𝑥(𝑥 − 2)
Example 2. Factor (2𝑥 + 4)(𝑥 − 3) − 5(𝑥 − 3)
Solution: The two terms have the common factor 𝑥 − 3.
(2𝑥 + 4)(𝑥 − 3) − 5(𝑥 − 3) = [(2𝑥 + 4) − 5](𝑥 − 3) Distributive
Property
= (2𝑥 − 1)(𝑥 − 3) Simplify
Example 3. Factor 8𝑥 𝑦 + 6𝑥 𝑦 − 2𝑥𝑦
4 2 3 3 4

Solution: Given that


8, 6, and −2 have the GCF of 2
𝑥4, 𝑥3, and 𝑥 have the GCF of 𝑥
have the GCF of 𝑦2
So the GCF of the three terms in the polynomial is 2𝑥𝑦2, and we have
8𝑥4𝑦2 + 6𝑥3𝑦3 − 2𝑥𝑦4 = 2𝑥𝑦2(4𝑥3) + 2𝑥𝑦2(3𝑥2𝑦) + 2𝑥𝑦2(−𝑦2)
= 2𝑥𝑦2(4𝑥3 + 3𝑥2𝑦 − 𝑦2)
 Factoring Trinomials
 To factor a trinomial of the form 𝑥2 + 𝑏𝑥 + 𝑐, we note that
(𝑥 + 𝑟)(𝑥 + 𝑠) = 𝑥2 + (𝑟 + 𝑠)𝑥 + 𝑟𝑠
So we need to choose numbers 𝑟 and 𝑠 so that 𝑟 + 𝑠 = 𝑏 and 𝑟𝑠 = 𝑐.

27
Prepared by: Rennil S. Bornia MST-Math
Self- Instructional Module on College and Advance Algebra & Advanced Mathematics

Points to Remember to Factor 𝒂𝒙𝟐 + 𝒃𝒙 + 𝒄, 𝒂 > 𝟎

1.The constant term of the trinomial is the product of the constant terms of the
binomials.
2.The coefficient in the trinomial is the sum of the constant terms of the binomials.
3.If the constant term of the trinomial is positive, the constant terms of the binomials
have the same sign as the coefficient in the trinomial.
4.If the constant term of the trinomial is negative, the constant terms of the
binomials have opposite signs.
5.If the terms of the trinomial do not have a common factor, then neither binomial
will have a common factor.

Example 4. Factor 𝑥2 + 7𝑥 + 12
Solution: Find two integers whose product is 12 and whose sum is 7. By trial and error,
we find that the two integers are 3 and 4. Thus the factorization is
𝑥2 + 7𝑥 + 12 = (𝑥 + 3)(𝑥 + 4)
Example 5. Factor 6𝑥2 + 7𝑥 − 5
Solution: We can factor 6 as 6 ∙ 1 or 3 ∙ 2, and −5 as −5 ∙ 1 or 5 ∙ (−1). By trying these
possibilities, we arrive at the factorization
6𝑥2 + 7𝑥 − 5 = (3𝑥 + 5)(2𝑥 − 1)

Sometimes it is impossible to factor a polynomial into the product of two polynomials


having integer coefficients. Such polynomials are said to be non-factorable over the
integers. To determine whether the trinomial is factorable or non-factorable, you may
use Factorization Theorem. It will indicate whether the trinomial is factorable over the
integers.

Factorization Theorem
The trinomial with integer coefficients and can be factored 𝑎𝑥2 + 𝑏𝑥 + 𝑐, with integer
coefficients 𝑎, 𝑏, and 𝑐 as the product of two binomials with integer coefficients if and
only if 𝑏2 − 4𝑎𝑐 is a perfect square.

Example 6. Determine whether each trinomial is factorable over the integers.


a. 4𝑥2 + 8𝑥 − 7 b. 6𝑥2 − 5𝑥 − 4
Solution:
a. The coefficients of 4𝑥2 + 8𝑥 − 7 are 𝑎 = 4, 𝑏 = 8, and 𝑐 = −7. Applying the factorization
theorem yields 𝑏2 − 4𝑎𝑐 = 82 − 4(4)(−7) = 176
Because 176 is not a perfect square, the trinomial is non-factorable over the
integers. b. The coefficients of 6𝑥2 − 5𝑥 − 4 are 𝑎 = 6, 𝑏 = −5, and 𝑐 = −4. Thus
𝑏2 − 4𝑎𝑐 = (−5)2 − 4(6)(−4) = 121

28
Prepared by: Rennil S. Bornia MST-Math
Self- Instructional Module on College and Advance Algebra & Advanced Mathematics

Because 121 is a perfect square, the trinomial is factorable over the integers.
Using the methods that we have developed, we find 6𝑥2 − 5𝑥 − 4 = (3𝑥 − 4)
(2𝑥 + 1)

 Special Factoring Formulas


 The following formulas can be used to factor some special algebraic expressions.

FACTORING FORMULAS
1. 𝑎2 − 𝑏2 = (𝑎 − 𝑏)(𝑎 + 𝑏) Difference of Two Perfect Squares
2. 𝑎2 + 2𝑎𝑏 + 𝑏2 = (𝑎 + 𝑏)2 Perfect Square Trinomial

3. 𝑎2 − 2𝑎𝑏 + 𝑏2 = (𝑎 − 𝑏)2 Perfect Square Trinomial

4. 𝑎3 − 𝑏3 = (𝑎 − 𝑏)(𝑎2 + 𝑎𝑏 + 𝑏2) Difference of Two Perfect Cubes


5. 𝑎3 + 𝑏3 = (𝑎 + 𝑏)(𝑎2 − 𝑎𝑏 + 𝑏2) Sum of Two Perfect Cubes

Example 7. Factor 4𝑥2 − 25


Solution: Using the Difference of Two Perfect Squares Formula with 𝑎 = 2𝑥 and𝑏 = 5, we have
4𝑥2 − 25 = (2𝑥)2 − 52 = (2𝑥 − 5)(2𝑥 + 5)
Example 8. Factor (𝑥 + 𝑦)2 − 𝑧2
Solution: Use the Difference of Two Perfect Squares Formula with 𝑎 = 𝑥 + 𝑦 and 𝑏 = 𝑧.
(𝑥 + 𝑦)2 − 𝑧2 = (𝑥 + 𝑦 − 𝑧)(𝑥 + 𝑦 + 𝑧)
Example 9. Factor 27𝑥3 − 1
Solution: Using the Difference of Two Perfect Cubes Formula with 𝑎 = 3𝑥 and𝑏 = 1, we get
27𝑥3 − 1 = (3𝑥)3 − 13 = (3𝑥 − 1)[(3𝑥)2 + (3𝑥)(1) + 12]
= (3𝑥 − 1)(9𝑥2 + 3𝑥 + 1)
Example 10. Factor 𝑥6 + 8
Solution: Using the Sum of Two Perfect Cubes Formula with 𝑎 = 𝑥2 and 𝑏 = 2, we have
𝑥6 + 8 = (𝑥2)3 + 23 = (𝑥2 + 2)(𝑥4 − 2𝑥2 + 4)
Example 11. Factor 𝑥2 + 6𝑥 + 9
Solution: Here 𝑎 = 𝑥 and 𝑏 = 3, so . Since the middle term is 6𝑥, the trinomial is a
perfect square. By the Perfect Square Trinomial Formula, we have
𝑥2 + 6𝑥 + 9 = (𝑥 + 3)2
Example 12. Factor 4𝑥2 − 4𝑥𝑦 + 𝑦2
Solution: Here 𝑎 = 2𝑥 and 𝑏 = 𝑦, so . Since the middle term is −4𝑥𝑦, the trinomial is a
perfect square. By the Perfect Square Trinomial Formula, we have
4𝑥2 − 4𝑥𝑦 + 𝑦2 = (2𝑥 − 𝑦)2

• Factoring by Grouping Terms


29
Prepared by: Rennil S. Bornia MST-Math
Self- Instructional Module on College and Advance Algebra & Advanced Mathematics

 Polynomials with at least four terms can sometimes be factored by grouping


terms. The following example illustrates the idea.
Example 13. Factor each polynomial.
a. 𝑥3 + 𝑥2 + 4𝑥 + 4 b. 𝑥3 − 2𝑥2 − 3𝑥 + 6
Solution:
a. 𝑥3 + 𝑥2 + 4𝑥 + 4 = (𝑥3 + 𝑥2) + (4𝑥 + 4) Group terms
2
= 𝑥 (𝑥 + 1) + 4(𝑥 + 1) Factor out common factors
2
= (𝑥 + 4)(𝑥 + 1) Factor out 𝑥 + 1 from each
term
b. 𝑥 − 2𝑥 − 3𝑥 + 6 = (𝑥 − 2𝑥 ) − (3𝑥 − 6)
3 2 3 2
Group terms
2
= 𝑥 (𝑥 − 2) − 3(𝑥 − 2) Factor out common factors
2
= (𝑥 − 3)(𝑥 − 2) Factor out 𝑥 − 1 from each
term

• Factoring an Expression Completely


 An expression is factored completely when we cannot factor it anymore.
 To factor an expression completely, factor out common factors first, then check
the result whether it can be factored further by using other methods of factoring.
Continue repeating this process until the expression is factored completely.

Example 14. Factor each expression completely.


a. 2𝑥4 − 8𝑥2
Solution: We first factor out the power of 𝑥 with the smallest exponent.
2𝑥4 − 8𝑥2 = 2𝑥2(𝑥2 − 4) Common factor is 2𝑥2
= 2𝑥2(𝑥 − 2)(𝑥 + 2) Factor 𝑥2 − 4 as a difference of squares b. 𝑥5𝑦2 − 𝑥𝑦6
Solution: We first factor out the powers of 𝑥 and 𝑦 with the smallest exponents.
𝑥5𝑦2 − 𝑥𝑦6 = 𝑥𝑦2(𝑥4 − 𝑦4) Common factor is 𝑥𝑦2
= 𝑥𝑦2(𝑥2 + 𝑦2)(𝑥2 − 𝑦2) Factor 𝑥4 − 𝑦4 as a difference of squares
= 𝑥𝑦2(𝑥2 + 𝑦2)(𝑥 + 𝑦)(𝑥 − 𝑦) Factor 𝑥2 − 𝑦2 as a difference of squares

D. Applications
Learning Task 2.2.3: Factoring Polynomials Flowchart
Instructions:
1. Make your own flowchart which illustrates a sequences of steps for factoring
polynomials.
2. Do not copy any available flowchart online.

30
Prepared by: Rennil S. Bornia MST-Math
Self- Instructional Module on College and Advance Algebra & Advanced Mathematics

IV. Assessment
Learning Task 2.2.4: Evaluation Quiz
A. Factor out the common factor from each B. Factor the trinomial.
polynomial. 6. 3𝑥2 − 16𝑥 + 5
1. −15𝑥2 − 12𝑥 7. 𝑥2 − 6𝑥 + 5
2 2
2. 10𝑥 + 6𝑥𝑦 − 14𝑥𝑦 8. 5𝑥2 − 7𝑥 − 6
3. 6𝑎3𝑏2 − 12𝑎2𝑏 + 72𝑎𝑏3 9. 2𝑥2 − 5𝑥 − 7
4. (𝑥 − 4)(2𝑎 − 𝑏) + (𝑥 + 4)(2𝑎 − 𝑏) 10. (3𝑥 + 2)2 + 8(3𝑥 + 2) + 12
2
5. 8𝑥 + 12𝑥 − 40
C. Use the factorization theorem to D. Factor each difference of squares over the
determine whether each trinomial is integers.
factorable over the integers. 16. 4𝑎2 − 9
11. 8𝑥2 + 26𝑥 + 15 17. 1 − 100𝑥2
12. 4𝑥2 − 5𝑥 + 6 18. (5𝑥 + 3)2 − 9
2
13. 6𝑥 − 14𝑥 + 5 19. 81𝑏2 − 16𝑐2
14. 16𝑥2 + 8𝑥 − 35 20. 𝑥2 − 64
15. 6𝑥2 + 8𝑥 − 3
E. Factor each perfect-square trinomial. F. Factor each sum or difference of cubes
21. 𝑥2 + 10𝑥 + 25 over the integers.
22. 𝑎2 − 14𝑎 + 49 26. 8𝑥3 − 27𝑦3
23. 4𝑥2 + 12𝑥 + 9 27. 8 − 𝑥6
24. 25𝑦2 + 40𝑦 + 16 28. (𝑥 − 2)3 − 1 29. 1 + 𝑦12
25. 9𝑥4 − 30𝑥2𝑦2 + 25𝑦4 30. (𝑦 + 3)3 + 8
G. Factor the expression by grouping terms. H. Factor the expression completely.
31. 𝑥3 + 4𝑥2 + 𝑥 + 4 36. 𝑥2(𝑥2 − 1) − 9(𝑥2 − 1)
32. 3𝑥3 − 𝑥2 + 6𝑥 − 2 37. 9𝑥2 − 36𝑥 − 45
33. −9𝑥3 − 3𝑥2 + 3𝑥 + 1 38. 18𝑦3𝑥2 − 2𝑥𝑦4
34. 18𝑤3 + 15𝑤2 + 12𝑤 + 10 39. 8𝑥3 − 125
35. 10𝑥3 − 15𝑧2 − 4𝑧 + 6 40. (𝑎2 + 2𝑎)2 − 2(𝑎2 + 2𝑎) − 3

V. Enhancement
Learning Task 2.2.5: Box Method Factoring
Instructions:

31
Prepared by: Rennil S. Bornia MST-Math
Self- Instructional Module on College and Advance Algebra & Advanced Mathematics

1. Watch YouTube video on factoring trinomials using box


method with the link https://www.youtube.com/watch?
v=PTW8UL0Bwyg. 2. Factor the following trinomials using box
method.
a. 9𝑥2 + 10𝑥 + 1 d. 6𝑥2 = 25𝑥 = 4
b. 8𝑥2 + 10𝑥𝑦 − 25𝑦2 e. 57𝑦2 + 𝑦 − 6
c. 8𝑎2 − 26𝑎 + 15
3. Show the step by step process of factoring trinomials using the said method.

VI. References
Ratti, J. M., & Marcus, S. (2014). College Algebra and Trigonometry (3rd Ed.). Pearson
Education Limited. Stewart, J., Redlin, L., & Watson, S. (2015). Algebra and trigonometry.
Cengage Learning
Lesson 2.3: Algebraic Fractions
I. Desired Learning Outcomes

At the end of the lesson, you will be able to:


• determine the domain of an algebraic fraction;
• perform the fundamental operations on algebraic fractions;
• simplify a complex fraction; and
• rationalize the denominator or numerator of an algebraic fraction.

II. Lesson Overview

Just like polynomials, algebraic fractions, also known as rational expressions, appear
frequently in Algebra and higher mathematics. That is why, you must understand how to
perform the basic operations with rational expressions. Moreover, algebraic fractions can
be useful tools for representing real life situations and for finding answers to real problems.
In particular, they are quite good for describing distance-speed-time questions, and
modeling multi-person work problems. Thus, this lesson provides you with an opportunity
to perform the fundamental operations on algebraic functions and rationalize numerator
and denominator of algebraic fractions.

III. Course Contents and Learning Experiences

A. Activity
Learning Task 2.3.1: Fraction Review
Instructions: Complete the following exercises.
1. Simplify: 4. Multiply:

2. Add: 5. Divide:

32
Prepared by: Rennil S. Bornia MST-Math
Self- Instructional Module on College and Advance Algebra & Advanced Mathematics

3. Subtract: 6. Simplify:
B. Analysis
Learning Task # 2.3.2: Think and Discuss
Instructions: Answer the following questions:
1. How did you simplify fractions?
2. How did you add or subtract fractions with common denominator?
3. How did you add or subtract fractions with no common denominator?
4. How did you multiply fractions?
5. How did you divide fractions?
6. How did you simplify complex fractions?
7. Explain the difference between an algebraic fraction and a fractional expression.

C. Abstraction

Key Concepts

 Algebraic Fractions – is a fraction whose numerator and denominator are algebraic


expressions. It is also known as rational expressions.

Examples:
¤ The Domain of an Algebraic Fractions
It is the set of all real numbers that can be used as replacements for the variable.
o Any value of the variable that causes division by zero is excluded from the domain
of the rational expression.

Steps to Find the Domain of Algebraic Fractions


1.Set the denominator equal to zero and solve the resulting equation.
2.The domain is the set of all real numbers excluding the values found in step 1.

 Example 1: Finding the Domain of an Expression Find the domains of the


following expressions.

Solution:
(a) 𝑥 − 3 = 0 𝑥=3
The denominator is zero when 𝑥 = 3. Since division by zero is not defined, we must
have 𝑥 ≠ 3. Thus the domain is all real numbers except 3. We can write this in set
notation as {𝑥|𝑥 ≠ 3}

(b) We first factor the denominator:


33
Prepared by: Rennil S. Bornia MST-Math
Self- Instructional Module on College and Advance Algebra & Advanced Mathematics

Since the denominator is zero when 𝑥 = 2 or 3, the expression is not


defined for these numbers. The domain is {𝑥│𝑥 ≠ 2 and 𝑥 ≠ 3}.
(c) For the numerator to be defined, we must have 𝑥 ≥ 0. Also, we cannot divide by
zero, so 𝑥 ≠ 5. Thus the domain is {𝑥│𝑥 ≥ 0 and 𝑥 ≠ 5}.

¤ Simplifying Algebraic Fractions


To simplify algebraic fractions, we factor both numerator and denominator and use the
following property of fractions:
𝐴𝐶 𝐴
=
𝐵𝐶 𝐵
This allows us to cancel common factors from the numerator and denominator.

 Example 2. Simplifying Algebraic Fractions by Cancellation


𝑥2−1
Simplify:
2

Solution: Factor
𝑥+1
Cancel common factor: (𝑥 − 1)
¤ Multiplying Algebraic Fractions
To multiply algebraic fractions, we use the following property of fractions:

𝐴 𝐶 𝐴𝐶 This says that to multiply two fractions, we multiply their


∙ =
𝐵 𝐷 𝐵𝐷 numerators and multiply their denominators.

 Example 3: Multiplying Algebraic Fractions


2
Perform the indicated
multiplication and simplify:
2

Solution: Factor

Property of fractions

Cancel common factors: (𝑥 − 1)and (𝑥 + 4)

¤ Dividing Algebraic Fractions


To divide algebraic fractions, we use the following property of fractions:

𝐴 𝐶 𝐴 𝐷
÷ = ∙
𝐵 𝐷 𝐵 𝐶

34
Prepared by: Rennil S. Bornia MST-Math
Self- Instructional Module on College and Advance Algebra & Advanced Mathematics

This says that to divide a fraction by another fraction, we invert the divisor and multiply.

 Example 4: Dividing Rational Expressions


2
Perform the indicated division and simplify:
2 2

Solution: Invert divisor and multiply

Factor
Cancel common factors: (𝑥 − 4) and (𝑥 + 2)

¤ Adding and Subtracting Algebraic Fractions


To add or subtract algebraic fractions, we first find a common denominator and then use
the following property of fractions:
𝐴 𝐵 𝐴+𝐵
+ =
𝐶 𝐶 𝐶

The following procedure can be used to determine the least common denominator
(LCD) of rational expressions. It is similar to the process used to find the LCD of
rational numbers.

 Determining the LCD of Algebraic Fractions


1. Factor each denominator completely and express repeated factors using
exponential notation. 2. Identify the largest power of each factor in any single
factorization. The LCD is the product of each factor raised to its largest power.

 Example 5. The algebraic fractions have an LCD of (𝑥 + 3)(2𝑥 − 1).


5𝑥 7
 Example 6. The algebraic fractions and
have an LCD of 𝑥(𝑥 + 5) (𝑥 − 7) .
2 3

 Example 7: Adding and Subtracting Rational Expressions Perform the indicated


operations and simplify.

Solution:
(a) Here the LCD is simply the product (𝑥 − 1)(𝑥 +
2).
3 3 2 1
Write fractions using LCD

Add fractions

35
Prepared by: Rennil S. Bornia MST-Math
Self- Instructional Module on College and Advance Algebra & Advanced Mathematics

Combine terms in numerator


(b) Factor the denominators:
𝑥2 − 3𝑥 − 10 = (𝑥 − 5)(𝑥 + 2)
𝑥2 − 7𝑥 + 10 = (𝑥 − 5)(𝑥 − 2)
The LCD is (𝑥 − 5)(𝑥 + 2)(𝑥 − 2). Write equivalent fractions in terms of the
LCD, and then subtract.

¤ Complex Fractions
A complex fraction is a fraction whose numerator or denominator contains one or more
fractions. Simplify complex fractions using one of the following methods.

Methods for Simplifying Complex Fractions


Method 1: Multiply by 1 in the form LCD/LCD.
1.
Determine the LCD of all fractions in the complex fraction.

2.
Multiply both the numerator and the denominator of the complex fraction by the LCD.
3.
If possible, simplify the resulting algebraic fractions.
Method 2: Multiply the numerator by the reciprocal of the denominator.

1. Simplify the numerator to a single fraction and the denominator to a single fraction.

2. Using the definition for dividing fractions, multiply the numerator by the reciprocal of
the denominator.

3. If possible, simplify the resulting algebraic fractions.


 Example 8: Simplify Complex Fractions

Simplify: (a)

Solution:

36
Prepared by: Rennil S. Bornia MST-Math
Self- Instructional Module on College and Advance Algebra & Advanced Mathematics

(a) Simplify the numerator to a single (b) Multiply the numerator and
denominator fraction and the denominator to a single by the LCD of all
the fractions.
fraction.

Simplify.

Subtract. The LCD is 𝑥 + 2.

¤ Rationalizing the Denominator or the Numerator


If a fraction has a denominator of the form , we may rationalize the denominator
by multiplying numerator and denominator by the conjugate radical .

 Example 9. Rationalize the numerator:


Solution: We multiply both the numerator and the denominator by the conjugate radical

Multiply numerator and denominator by the conjugate radical

Use Product Formula (𝑎 + 𝑏)(𝑎 − 𝑏) = 𝑎2 − 𝑏2

Cancel common factors

37
Prepared by: Rennil S. Bornia MST-Math
Self- Instructional Module on College and Advance Algebra & Advanced Mathematics

1
 Example 10: Rationalize the denominator:
Solution: We multiply both the numerator and the denominator by the conjugate radical of

1 + √2, which is 1 − √2.

Multiply numerator and denominator by the conjugate radical

Use Product Formula (𝑎 + 𝑏)(𝑎 − 𝑏) = 𝑎2 − 𝑏2

D. Applications
Learning Task 2.3.3: Think-Pair-Explain
Instructions: In a group of 3, read and understand each problem or question. Then,
explain your answer.
1. Aaron multiplied (equal to 1) to obtain the fraction . Is the fraction equal to

the fraction
𝑏𝑏

for all values of 𝑏? Explain your answer.


2. Kevin used cancellation to reduce to lowest terms as shown below. What is the error in
Kevin’s work? 𝑎+4
𝑎+8

𝑎 +4 2
3. Kevin let 𝑎 = 4 to prove that when reduced to lowest = 3 . Explain to Kevin why his
𝑎 +8terms,
reasoning is incorrect.
4. When reduced to lowest terms, a fraction whose numerator is 𝑥2 − 3𝑥 + 2 equals −1. What
is the denominator of the fraction? Explain your answer.

5. Does for all values of 𝑥 and 𝑧? Explain your answer.


6. Explain why the quotient is undefined for 𝑥 = 2 and for 𝑥 = 3.

7. To find the quotient , Ruth cancelled (𝑥 − 4) in the numerator and denominator


and wrote
. Is Ruth’s answer correct? Explain why or why not.
8. In the expression . Can we divide 2𝑥 and 6𝑥 by 2 to write the answer in lowest terms as
? Explain why or why not.
9. Joey said that . Do you agree with Joey? Explain why or why not.

38
Prepared by: Rennil S. Bornia MST-Math
Self- Instructional Module on College and Advance Algebra & Advanced Mathematics

IV. Assessment
Learning Task 2.3.4: Evaluation Quiz
A. Reduce each algebraic fractions to lowest terms. Specify the domain of the algebraic
fraction by identifying all real numbers that must be excluded from the domain.

1.

6.
B. Perform the indicated operations and simplify the
result. Leave the numerator and denominator in your
answer in factored form.

C. Simplify each complex fraction.

D. Rationalize denominator or numerator.

V. Enhancement
Learning Task 2.3.5: Student Learning Map
Instructions: Create a learning map regarding the things that you have learned in Unit 2
which is algebraic expressions. A learning map is a “graphic organizer that highlights the
knowledge, skills, and big ideas that students get from a lesson, unit, or course. In creating
a learning map, you may follow the following steps:
Step 1 - Write down all the discrete knowledge, skills, and big ideas for a unit.

39
Prepared by: Rennil S. Bornia MST-Math
Self- Instructional Module on College and Advance Algebra & Advanced Mathematics

Step 2 – Organize the notes into clusters of knowledge, skills, and big ideas that make
sense to teach together or in sequence.
Step 3 – Articulate the connections between the various elements using labels.

VI. References
Ratti, J. M., & Marcus, S. (2014). College Algebra and Trigonometry (3rd Ed.). Pearson
Education Limited.
Stewart, J., Redlin, L., & Watson, S. (2015). Algebra and trigonometry. Cengage Learning.
http://www.hufsd.edu/assets/pdfs/academics/algebra_text/Chapter14.pdf

UNIT 3: EQUATIONS AND INEQUALITIES


Unit Overview
We use equations and inequalities very frequently
in daily life. Equations and inequalities are useful
in analyzing designs found in the art and
architecture of ancient civilizations, in
understanding the dazzling geometric
constructions that abound in nature, and in solving
problems common to industry and daily life.

Today’s computer chips used in all the machines


we use in daily routine like washers, dryers, cars,
backs, etc. All chips, which we use in these machines based on mathematical equations
and algorithms. Mathematical equations are also used in traffic control, aircraft, space
program and medicine and so on. So we should always remember that any math equation
result has the potential to change the world. That is the reason all mathematical equations
are important in our life.

Hence, this unit provides you with lessons and various examples regarding solving of
equations and inequalities. Lesson 3.1 enables you to classify equations and find the
solution set of linear and quadratic equations. Also, in Lesson 3.2, you will learn how to
solve other types of equations such as polynomials, rational, radical, and equation with
rational exponents. Lastly, in Lesson 3.3 you will solve and graph inequalities.

Moreover, at the end of this unit, you should be able to:


 solve linear equation;
 use linear equations to solve applied problems;

40
Prepared by: Rennil S. Bornia MST-Math
Self- Instructional Module on College and Advance Algebra & Advanced Mathematics

 solve quadratic equations by factoring, completing the square, and using quadratic
formula;
 solve other types of equations such as polynomials, rational, radical, equation with
rational exponents, and equation that are quadratic in form;
 solve and graph linear inequalities and non-linear inequalities.

Lesson 3.1: Solving Linear and Quadratic Equations


I. Desired Learning Outcomes

At the end of the lesson, you will be able to:


• differentiate expression and equality;
• solve linear equations;
• use linear equations to solve applied problems;
• solve quadratic equations by factoring;
• solve quadratic equations by completing the square;
• solve quadratic equations by using quadratic formula; and
• use the discriminant to determine the number of real solutions of the equation.

II. Lesson Overview

The applications of algebra often lead to equations. The skills that you learned in Unit 2,
such as combining like terms and performing operations with algebraic expressions, will
now be used to solve equations. Equations are the basic mathematical tool for solving real-
world problems. In this lesson, you will learn how to solve equations, as well as how to
construct equations that model real-world situations

III. Course Contents and Learning Experiences

A.Activity
Learning Task 3.1.1: Solving Equations Maze

41
Prepared by: Rennil S. Bornia MST-Math
Self- Instructional Module on College and Advance Algebra & Advanced Mathematics

B.Analysis
Learning Task # 3.1.2: Think and Discuss
Instructions: Answer the following:

1.How did you solve for 𝑥?


2.Based on your solutions in solving the maze, identify the similarities and differences
between expressions and equations by using the Venn diagram.

C. Abstraction

Key Concepts

• Equation – is a statement about the equality of two algebraic expressions.


Example: 2𝑥 + 1 = 7
• Solution Set – is the set of all solutions to an equation.
• Solving the equation – is the process of finding the solution set of an equation.
• Equivalent equations – are two equations that have exactly the same solution or
solutions.
The process of solving an equation is often accomplished by producing a sequence of
equivalent equations until we arrive at an equation or equations of the form

IDENTIFYING TYPES OF EQUATIONS


1. An equation that is satisfied by all values in the domain of the variable is an
identity. For example, 2(𝑥 − 1) = 2𝑥 − 2. When you try to solve an identity, you
get a true statement such as 3 = 3 or 0 = 0.
42
Prepared by: Rennil S. Bornia MST-Math
Self- Instructional Module on College and Advance Algebra & Advanced Mathematics

2. An equation that is not an identity but is satisfied by at least one number in the
domain of the variable is a conditional equation. For example, 2𝑥 = 6 is a linear
conditional equation; 𝑥 = 3 is a solution, but 𝑥 = 0 is not.
3. 3. An equation that is not satisfied by any value of the variable is an inconsistent
equation. For example, 𝑥 = 𝑥 + 2 is an inconsistent equation. Because no number
is 2 more than itself, the solution set of the equation 𝑥 = 𝑥 + 2 is ∅. When you try to
solve an inconsistent equation, you obtain a false statement such as 0 = 2.

Properties of Equality
Addition and Subtraction Property of Equality
Adding or subtracting the same number to both sides of an equation does not change the
solution set to the equation. In symbols, if 𝑎 = 𝑏, then 𝑎 + 𝑐 = 𝑏 + 𝑐.

Multiplication and Division Property of Equality


Multiplying or dividing both sides of an equation by the same nonzero number does not change

the solution set to the equation. In symbols, if 𝑎 = 𝑏 and 𝑐 ≠ 0, then 𝑐𝑎 = 𝑐𝑏.

¤ Linear Equations

Definition of a Linear Equation

A linear equation, or first-degree equation, in the single variable is an equation


that can be written in the form 𝑎𝑥 + 𝑏 ≠ 0, where 𝑎 and 𝑏 are real numbers, with 𝑎 ≠ 0.

¤ Solving Linear Equations

Strategy for Solving a Conditional Linear Equation

1. If fractions are present, multiply each side by the LCD to eliminate them. If
decimals are
present, multiply each side by a power of 10 to eliminate them.
2. Use the distributive property to remove parentheses.
3. Combine any like terms.
4. Use the addition property of equality to get all variables on one side and numbers
on the other side.
5. Use the multiplication property of equality to get a single variable on one side.
6. Check by replacing the variable in the original equation with your solution.
43
Prepared by: Rennil S. Bornia MST-Math
Self- Instructional Module on College and Advance Algebra & Advanced Mathematics

Example 1. Solving Conditional Linear Equation


Solve the equation 7𝑥 − 4 = 3𝑥 + 8
Solution. We solve this by changing it to an equivalent equation with all terms that have the variable x
on one side and all constant terms on the other:
7𝑥 − 4 = 3𝑥 + 8 Given
equation
(7𝑥 − 4) + 4 = (3𝑥 + 8) + 4 Add 4
7𝑥 = 3𝑥 + 12 Simplify
7𝑥 − 3𝑥 = (3𝑥 + 12) − 3𝑥 Subtract 3𝑥
4𝑥 = 12 Simplify
Multiply by
𝑥=3 Simplify

When a linear equation involves fractions, solving the equation is usually easier if we first
multiply each side by the lowest common denominator (LCD) of the fractions, as we see in
the following examples. Example 2. Solving an Equation That Involves Fractions
Solve the equation .
Solution. The LCD of the denominators 6, 3, and 4 is 12, so we first multiply each side of the equation
by 12 to clear the denominators:
𝑥 2 3 Multiply by LCD
12 ∙ ( + ) = 12 ∙ 𝑥
6 3 4
2𝑥 + 8 = 9𝑥 Distributive Property
8 = 7𝑥 Subtract 2𝑥
Divide by 7.
Example 3: Solving a Linear Equation That Is
Inconsistent Solve 5𝑥 − [3𝑥 + 2(𝑥 − 1)] = 1.
Solution
5𝑥 − [3𝑥 + 2(𝑥 − 1)] = 1 Original equation
5𝑥 − [3𝑥 + 2𝑥 − 2] = 1 Distributive
Property
5𝑥 − 3𝑥 − 2𝑥 + 2 = 1 Distributive
Property
2=1 Combine like
terms
Because 2 = 1 is false, no number satisfies this equation. It is inconsistent.

Example 4: Solving an Equation That Is an Identity


Solve 1 − 5𝑦 + 2(𝑦 + 7) = 2𝑦 + 5(3 − 𝑦)
Solution:

44
Prepared by: Rennil S. Bornia MST-Math
Self- Instructional Module on College and Advance Algebra & Advanced Mathematics

1 − 5𝑦 + 2(𝑦 + 7) = 2𝑦 + 5(3 − 𝑦) Original equation


1 − 5𝑦 + 2𝑦 + 14 = 2𝑦 + 15 − 5𝑦 Distributive
Property
15 − 3𝑦 = 15 − 3𝑦 Simplify
15 − 3𝑦 + 3𝑦 = 15 − 3𝑦 + 3𝑦 Add 3y to both
sides
15 = 15 Simplify
We have shown that 15 = 15 is equivalent to the original equation. The equation 15 =
15 is always true; its solution set is the set of all real numbers. Therefore, the solution set of
the original equation is also the set of all real numbers. Thus, the original equation is an
identity and the solution set is (−∞, ∞). ¤ Quadratic Equations

QUADRATIC EQUATIONS

A quadratic equation is an equation of the form


𝒂𝒙𝟐 + 𝒃𝒙 + 𝒄 = 𝟎
where a, b, and c are real numbers with 𝑎 ≠ 0.

¤ Solving Quadratic Equations by Factoring

ZERO-PRODUCT PROPERTY

𝐴𝐵 = 0 if and only if 𝐴 = 0 or 𝐵 = 0

This means that if we can factor the left-hand side of a quadratic (or other) equation,
then we can solve it by setting each factor equal to 0 in turn. This method works only
when the right-hand side of the equation is 0.
Example 5. Solving a Quadratic Equation by Factoring
Solve the equation 𝑥2 + 5𝑥 = 24
Solution. We must first rewrite the equation so that the right-hand side is 0:
𝑥2 + 5𝑥 = 24 Given equation
2
𝑥 + 5𝑥 − 24 = 0 Subtract 24
(𝑥 − 3)(𝑥 + 8) = 0 Factor
𝑥−3=0 or 𝑥+8=0 Zero-Product
Property
𝑥=3 𝑥 = −8 Solve
The solutions are 𝑥 = 3 and 𝑥 = −8

¤ Solving Quadratic Equations by Completing the Square

45
Prepared by: Rennil S. Bornia MST-Math
Self- Instructional Module on College and Advance Algebra & Advanced Mathematics

COMPLETING THE SQUARE


2 𝑏 2
( 2 ) , the square of half the
To make𝑥 + 𝑏𝑥 a perfect square, add
𝑏 2 𝑏 2
coefficient of𝑥 . This gives the perfect square𝑥 2 + 𝑏𝑥 + ( 2 ) = (𝑥 + 2 )

To complete the square, we add a constant to a quadratic expression to make it a


perfect square. For

example, to make 𝑥2 + 6𝑥 a perfect square, we must add . Then 𝑥2 + 6𝑥 + 9 = (𝑥


2
+ 3) is a perfect square. The table gives some more examples of completing the
square.
Expression Add Complete the square

𝑥2 + 8𝑥 𝑥2 + 8𝑥 + 16 = (𝑥 + 4)2

𝑥2 − 12𝑥 𝑥2 − 12𝑥 + 36 = (𝑥 − 6)2

𝑥2 + 3𝑥

Example 6. Solving Quadratic Equations by Completing the


Square Solve each equation.
2
(a) 𝑥 − 8𝑥 + 13 = 0 (b) 3𝑥2 − 12𝑥 + 6 = 0
Solution
(a) 𝑥2 − 8𝑥 + 13 = 0 Given equation
𝑥2 − 8𝑥 = −13 Subtract 13
2 2
𝑥 − 8𝑥 + 16 = −13 + 16 8
Complete the square: add
(𝑥 − 4)2 = 3 Perfect square
𝑥 − 4 = ±√3 Take square root
𝑥 = 4 ± √3 Add 4

(b) After subtracting 6 from each side of the equation, we must factor the coefficient of 𝑥2
(the 3) from the left side to put the equation in the correct form for completing the
square:
3𝑥2 − 12𝑥 + 6 = 0 Given equation
2
3𝑥 − 12𝑥 = −6 Subtract 6
2
3(𝑥 − 4𝑥) = −6 Factor 3 from left hand side
Now we complete the square by adding (−2)2 = 4 inside the parentheses. Since
everything inside the parentheses is multiplied by 3, this means that we are actually
adding 3 ∙ 4 = 12 to the left side of the equation. Thus we must add 12 to the right
side as well:
46
Prepared by: Rennil S. Bornia MST-Math
Self- Instructional Module on College and Advance Algebra & Advanced Mathematics

Complete the square: add 4


Perfect square
Divide by 3
Take square root
Add 2
¤ The Quadratic Formula
We can use the technique of completing the square to derive a formula for the
roots of the general quadratic equation 𝑎𝑥2 + 𝑏𝑥 + 𝑐 = 0.

THE QUADRATIC FORMULA


𝑎𝑥 2 + 𝑏𝑥 + 𝑐 = 0,
The roots of the quadratic equation
−𝑏±√𝑏 2 −4𝑎𝑐
where𝑎 ≠ 0, are 𝑥 = 2𝑎

Example 7. Find all solutions of each equation.


(a) 3𝑥2 − 5𝑥 − 1 = 0 (b) 4𝑥2 + 12𝑥 + 9 = 0 (c) 𝑥2 + 2𝑥 + 2 = 0
Solution
(a) In this quadratic equation 𝑎 = 3, 𝑏 = −5, and 𝑐 = −1:
−(−5)±√(−5)2 −4(3)(−1) 5±√37
By the Quadratic Formula,
𝑥= 2(3)
= 6

This equation has two solutions,


(b) Using the Quadratic Formula with 𝑎 = 4, 𝑏 =
12, and 𝑐 = 2 gives

This equation has only one solution, .

(c) Using the Quadratic Formula with 𝑎 = 1, 𝑏 = 2, and 𝑐 = 2 gives

Since the square of any real numbers is nonnegative, √−1 is undefined in the
real number system. The equation has no real solution.

¤ The Discriminant
Discriminant of the quadratic equation – is the quantity 𝑏2 − 4𝑎𝑐 in the Quadratic
Formula.

THE DISCRIMINANT

The discriminant of the quadratic equation 𝑎𝑥2 + 𝑏𝑥 + 𝑐 = 0, where


𝑎 ≠ 0 is 𝐷 = 𝑏2 − 4𝑎𝑐.

47
Prepared by: Rennil S. Bornia MST-Math
Self- Instructional Module on College and Advance Algebra & Advanced Mathematics

1. If 𝐷 > 0, then the equation has two distinct real solutions.

2. If 𝐷 = 0, then the equation has exactly one real solution.

3. If 𝐷 < 0, then the equation has no real solution.

Example 8. Using the Discriminant


Determine the number of solutions each quadratic equation has by using the discriminant.
(a) 𝑥2 + 4𝑥 − 1 = 0 (b) 4𝑥2 − 12𝑥 + 9 = 0
Solution
(a) The equation has two distinct real solutions since the discriminant 𝐷 > 0.
𝐷 = 42 − 4(1)(−1) = 20
(b) The equation has two distinct real solutions since the discriminant .

(c) The equation has two distinct real solutions since the discriminant .

D. Applications
Learning Task 3.1.3: Think-Pair-Solve
Instructions: Find a pair and solve the following problems applying the procedures in
solving linear equations.
1. The sum of three consecutive even integers is Find the integers.
2. The perimeter of a rectangular closet is 25 feet. What are the length and width of the
rectangular closet if the length is 4 feet longer than the width?
3. John won on a lottery. He decided to keep a certain amount for himself, give
one-half the amount he kept for himself to his daughter, and give one-fourth the amount
he kept for himself to his dad. How much did each person get?
4. The total amount of the real estate is 11,250,000. It will be divided between two
daughters so that the older daughter receives four times as much as the younger
daughter. Find each daughter’s share of the estate.
5. Mr. Mendoza received an inheritance of 350,000. He put part of it in a tax shelter
paying 9% interest and part in a bank paying 6%. If his annual interest totals 27,000,
how much did he invest at each rate?
6. The number of girls in the class is 3/5 of the boys. Find the number of boys and girls in
the class if there are a total of 42 students in the class.
7. The cost of a ball pen is 10 more than the cost of a pencil. If the cost of 6 ball pens
and 8 pencils is 640, find the cost of each.
8. Mother’s age is three times her daughter’s age. Four years ago, she was 4 times her
daughter’s age. Find their present ages.
9. Among the two supplementary angles, the measure of the smaller angle is 25° less than
the measure of the larger angle. Find their measures.
48
Prepared by: Rennil S. Bornia MST-Math
Self- Instructional Module on College and Advance Algebra & Advanced Mathematics

10. Three numbers are in the ratio and their sum is 375. Find the numbers.

IV. Assessment

Learning Task 3.1.4: Evaluation Quiz

Solve each linear equation.


1.
2.
3.

Solve the equation by factoring.


6. 9.
7. 10.
8.

Solve the equation by completing the square.


11. 𝑥2 + 2𝑥 − 5 14. 2𝑥2 + 8𝑥 + 1 = 0 12. 𝑥2 − 4𝑥 + 2 = 0
15. 𝑥 − 6𝑥 − 11 = 0 13. 4𝑥 − 𝑥 = 0
2 2

Solve the equation using the quadratic formula.


16. 𝑥2 + 2𝑥 − 4 = 0 19. 𝑥2 + 3𝑥 + 2 = 0 17. 𝑥2 − 7 = 4𝑥
2 2
20. 3𝑥 − 2𝑥 = 7 18. 6𝑥 = 7𝑥 + 5

Use the discriminant to determine the number of real solutions of the equation. Do not solve
the equation.
21. 𝑥2 − 6𝑥 + 1 = 0 22. 𝑥2 = 6𝑥 − 9
25. 𝑥2 + 2.20𝑥 + 1.21 = 0

V. Enhancement
Learning Task 3.1.5: Creating an Equation Sudoku Puzzle
Instructions: Create a Sudoku Puzzle in solving linear
equations.
You may refer to the following links for a sample copy. https://www.signnow.com/fill-and-sign-
pdf-form/32690-solving-linear-equations-sudoku-form http://www.hotelsrate.org/solving-linear-
equations-sudoku-answers/
Note: Do not copy any material available in the internet. Just use it for reference and guide.

49
Prepared by: Rennil S. Bornia MST-Math
Self- Instructional Module on College and Advance Algebra & Advanced Mathematics

VI. References
Ratti, J. M., & Marcus, S. (2014). College Algebra and Trigonometry (3rd Ed.). Pearson
Education Limited. Stewart, J., Redlin, L., & Watson, S. (2015). Algebra and trigonometry.
Cengage Learning.

Lesson 3.2: Solving Other Types of Equations

I. Desired Learning Outcomes

At the end of the lesson, you will be able to:


• solve equations by factoring;
• solve rational equations;
• solve equations involving radicals;
• solve equations with rational exponents; and  solve equations that are quadratic
in form.

II. Lesson Overview

In the previous lesson, you have learned how to solve linear and quadratic equation. In
this lesson, you will be given an opportunity to solve equations by factoring, solving rational
equations, radical equations, equations with rational exponents and equations that are
quadratic in form.

III. Course Contents and Learning Experiences

A.Activity
Learning Task 3.2.1: Warm-Up Review

Factor each expression by grouping.

50
Prepared by: Rennil S. Bornia MST-Math
Self- Instructional Module on College and Advance Algebra & Advanced Mathematics

1. 𝑥3 − 𝑥2 − 9𝑥 + 9 2. −𝑥3 − 2𝑥2 + 25𝑥 + 50


Find the LCD for each group of rational expressions

Simplify each expression.


9. 272⁄3 10. 45⁄2
11. 84⁄3 12. 253⁄2

15. ((3𝑥2 + 7)1⁄3)3 16. ((5𝑥 − 2)3⁄5)5⁄3

B.Analysis
Learning Task # 3.2.2: Think and Discuss
Instructions:
1. Compare rational equations, radical equations, and equations with rational exponents.
2. How are radicals and rational exponents related?
3. How did you find LCD of rational expressions?
4. How did you simplify number with rational exponents?
5. How did you simplify radical expressions?

C. Abstraction

¤ Solving Equations by Factoring

STRATEGY FOR SOLVING EQUATIONS BY FACTORING


Step 1 Make one side zero. Move all nonzero terms in the equation to one side (say, the left
side) so that the other side (the right side) is 0.
Step 2 Factor the left side.
Step 3 Use the zero-product property. Set each factor in Step 2 equal to 0 and then solve
the resulting equations.
Step 4 Check your solutions.

Example 1. Solving an Equation by Factoring


Solve by factoring: 𝑥4 = 9𝑥2
Solution:
Step 1 Move all terms to the left side.
𝑥4 = 9𝑥2 Original equation
𝑥4 − 9𝑥2 = 0 Subtract 9𝑥2 from both
Step 2 Factor. sides

51
Prepared by: Rennil S. Bornia MST-Math
Self- Instructional Module on College and Advance Algebra & Advanced Mathematics

𝑥2(𝑥2 − 9) = 0 𝑥4 − 9𝑥2 = 𝑥2(𝑥2 − 9)


𝑥2(𝑥 + 3)(𝑥 − 3) = 0 𝑥2 − 9 = (𝑥 + 3)(𝑥 − 3)
Step 3 Set each factor equal to zero.
𝑥2 = 0 𝑥 + 3 = 0 𝑥 − 3 = 0 Zero-product property
𝑥=0 𝑥 = −3 𝑥=3 Solve each equation for 𝑥.
Step 4 Check:
Let 𝑥 = 0 Let 𝑥 = −3 Let 𝑥 = 3
(0) 9(0)2
4
(−3)4
9(−3)2 (3)4 9(3)2
0 = 0 81 = 81 81 = 81
Thus, the solution set of the equation is {−3, 0, 3}. Note that 0 is a root of multiplicity 2.

¤ Rational Equations
• Recall that if at least one algebraic expression with the variable in the denominator
appears in an equation, then the equation is a rational equation.
• When we multiply a rational equation by an expression containing the variable, we
may introduce a solution that satisfies the new equation but does not satisfy the
original equation.
• Such a solution is called an extraneous solution or extraneous root. So
whenever we multiply an equation by an expression containing the variable, we
must check all possible solutions obtained to reject extraneous solutions (if any).

Example 2. Solving a Rational Equation


Solve:
Solution
Step 1 Find the LCD.
The LCD from the denominators 6, 𝑥 + 1, and 𝑥 is 6𝑥(𝑥 + 1).
Step 2 Multiply both sides of the equation by the LCD and make the right side 0.
Multiply both sides by the LCD.

Distributive property
𝑥(𝑥 + 1) + 6𝑥 = 6(𝑥 + 1) Simplify.
𝑥2 + 𝑥 + 6𝑥 = 6𝑥 + 6 Distributive property
𝑥2 + 𝑥 − 6 = 0 Subtract 6𝑥 + 6 from both
sides.
Step 3 Factor.
𝑥2 + 𝑥 − 6 = 0
(𝑥 + 3)(𝑥 − 2) = 0
Step 4 Set each factor equal to zero.

𝑥+3=0 𝑥−2=0

52
Prepared by: Rennil S. Bornia MST-Math
Self- Instructional Module on College and Advance Algebra & Advanced Mathematics

𝑥 = −3 𝑥=2
Step 5 Check. You should check the solutions, −3 and 2, in the original equation.
The solution set is {−3, 2}.

Example 3. Solving a Rational Equation with an Extraneous Solution


Solve:
Solution
Step 1 Find the LCD.
The LCD of the denominators (𝑥 − 1), (𝑥 + 1), (𝑥2 − 1) = (𝑥 − 1)(𝑥 = 1) is (𝑥 − 1)(𝑥 + 1).

Step 2

(𝑥 + 1)𝑥 − (𝑥 − 1) = 2𝑥
𝑥2 + 𝑥 − 𝑥 + 1 − 2𝑥 = 2𝑥 − 2𝑥
𝑥2 − 2𝑥 + 1 = 0
Step 3 (𝑥 − 1)(𝑥 − 1) = 0
Step 4 𝑥 − 1 = 0 or 𝑥=1
Step 5 Check: Substitute 𝑥 = 1 in the original equation.

The denominator of the first term is 0, so the equation is undefined for 𝑥 = 1.


Therefore, 𝑥 = 1 is not a solution of the original equation; it is extraneous. The original
equation has no solution.

¤ Equations Involving Radicals

SOLVING EQUATIONS CONTAINING SQUARE ROOTS

Step 1 Isolate one radical to one side of the equation.

Step 2 Square both sides of the equation in Step 1 and simplify.

Step 3 If the equation obtained in Step 2 contains a radical, repeat


Steps 1 and 2 to get an equation that is free of radicals.
Step 4 Solve the equation obtained in Steps 1–3.
Step 5 Check the solutions in the original equation.
Example 4: Solving an Equation Involving a Radical
Solve:
Solution:
Step 1 We begin by isolating the radical to one side of the equation.
Original equation
Subtract 1 from both sides.
53
Prepared by: Rennil S. Bornia MST-Math
Self- Instructional Module on College and Advance Algebra & Advanced Mathematics

Step 2 Next, we square both sides and simplify.

2𝑥 + 1 − (2𝑥 + 1) = 𝑥2 − 2𝑥 + 1 − (2𝑥 + 1) Subtract 2𝑥 + 1


2
0 = 𝑥 − 4𝑥 Simplify
0 = 𝑥(𝑥 − 4) Factor
Step 3 Set each factor equal to zero.
𝑥=0 𝑥−4=0
𝑥 = −3 𝑥=4
Step 4 Check
Let 𝑥 = 0 Let 𝑥 = 4
4 = 4

Thus, 0 is an extraneous root; the only


solution of the given equation is 4, and the solution set is
{4}.
Example 5: Solving an Equation Involving Two
Radicals
Solve:
Solution:
Step 1 We first isolate one of the radicals.
Original equation
√2𝑥 − 1 = 1 + √𝑥 − 1 Add to both sides to isolate the radical.
Step 2 Square both sides of the last equation.

2
2𝑥 − 1 = 1 + 2√𝑥 − 1 + 𝑥 − 1 ( √ 𝑎) = 𝑎
Simplify: use

Step 3 The last equation still contains a radical. We repeat the process
of isolating the radical expression.

𝑥 − 1 = 2√𝑥 − 1 Subtract 𝑥 from both sides to isolate the radical.


Square both sides
𝑥 − 2𝑥 + 1 = 4(𝑥 − 1)
2
Use (𝑎 − 𝑏)2 = 𝑎2 − 2𝑎𝑏 + 𝑏2 and (𝑎𝑏)2 = 𝑎2𝑏2
𝑥2 − 2𝑥 + 1 = 4𝑥 − 4 Distributive property

54
Prepared by: Rennil S. Bornia MST-Math
Self- Instructional Module on College and Advance Algebra & Advanced Mathematics

𝑥2 − 6𝑥 + 5 = 0 Add −4𝑥 − 4 to both sides; simplify


(𝑥 − 5)(𝑥 − 1) = 0 Factor.
Step 4 Set each factor equal to 0.
𝑥−5=0 𝑥−1=0
𝑥=5
𝑥=1
Step 5 Check: You should check that the solutions of the original equation are 1 and 5.
The solution set is {1, 5}.

¤ Equations with Rational Exponents


If a given equation can be expressed in the form 𝑢𝑚⁄𝑛 = 𝑘, then we isolate 𝑢 by raising
both sides to the power (the reciprocal of

Example 5. Solving Equations with Rational


Exponents

55
Prepared by: Rennil S. Bornia MST-Math
Self- Instructional Module on College and Advance Algebra & Advanced Mathematics

¤ Equations That Are Quadratic in Form


An equation in a variable 𝑥 is quadratic in form if it can be written as
𝑎𝑢2 + 𝑏𝑢 + 𝑐 = 0 (𝑎 ≠ 0),
where 𝑢 is an expression in the variable 𝑥. We solve the equation 𝑎𝑢2 + 𝑏𝑢 + 𝑐 = 0 for 𝑢.
Then the solutions of the original equation can be obtained by replacing u with the
expression in 𝑥 that 𝑢 represents. Example: Solving an Equation Quadratic in Form
Solve: 𝑥2⁄3 − 5𝑥1⁄3 + 6 = 0
Solution:

D. Applications
56
Prepared by: Rennil S. Bornia MST-Math
Self- Instructional Module on College and Advance Algebra & Advanced Mathematics

Learning Task 3.2.3: Solving an Equation in Different


Ways Instructions:
We have learned several different ways to solve an equation in this section. Some
equations can be tackled by more than one method. For example, the equation
is of quadratic type: We can solve it by letting and and 𝑥 = 𝑢2 and
factoring. Or we could solve for , square each side, and then solve the resulting
quadratic equation. Solve the following equations using both methods indicated, and show
that you get the same final answers.
1. Quadratic type; solve for the radical, and square
2. Quadratic type; multiply by LCD

IV. Assessment
Learning Task 3.2.4: Evaluation Quiz

Solve the equation by factoring.


1. 𝑥3 = 2𝑥2 4. 3𝑥4 = 24𝑥
2. 3𝑥4 − 27𝑥2 = 0 5. 𝑥3 − 36𝑥 = 16(𝑥 − 6)
3. 𝑥3 − 1 = 0

Solve each rational equation.

Solve each radical equation.

Solve each equation with rational


exponents.
16. (𝑥 − 4)3⁄2 = 27 19. (𝑥 + 7)3⁄2 = 64
17. (5𝑥 − 3)2⁄3 − 5 = 4 20. (2𝑥 − 6)2⁄3 + 9 = 13
18. 3(𝑥 − 2)3⁄5 + 4 = 7

Solve each equation that are quadratic in form.


24. 2𝑥1⁄2 + 3𝑥1⁄4 − 2 = 0
25. (3𝑡 + 1)2 − 3(3𝑡 + 1) + 2 = 0 23. 𝑥2⁄3 − 6𝑥1⁄3 + 8 = 0

57
Prepared by: Rennil S. Bornia MST-Math
Self- Instructional Module on College and Advance Algebra & Advanced Mathematics

V. Enhancement
Learning Task 3.2.5: Math Reflection: Great Equation
Race
Instructions: Write your reflection regarding your learnings and difficulties in solving linear
equations, quadratic equations, and other types of equations. Write 3 things that you have
learned, 2 things that were difficult, and 1 question that you still have. Lastly, try to answer
how will this help you in the future.

VI. References

Ratti, J. M., & Marcus, S. (2014). College Algebra and Trigonometry (3rd Ed.). Pearson
Education Limited. Stewart, J., Redlin, L., & Watson, S. (2015). Algebra and trigonometry.
Cengage Learning.

58
Prepared by: Rennil S. Bornia MST-Math
Self- Instructional Module on College and Advance Algebra & Advanced Mathematics

Lesson 3.3: Solving Inequalities


I. Desired Learning Outcomes

At the end of the lesson, the students must have:


• solve linear and non-linear inequalities;
• express the solution set of linear and non-linear inequalities in set-builder or
interval notation;
• solve problems involving inequalities; and
• graph the solution set of linear and non-linear inequalities;

II. Lesson Overview


The applications of algebra often lead to equations. The skills that you learned in Unit 2,
such as combining like terms and performing operations with algebraic expressions, will
now be used to solve equations. Equations are the basic mathematical tool for solving real-
world problems. In this lesson, we learn how to solve equations, as well as how to
construct equations that model real-world situations

III. Course Contents and Learning Experiences


A.Activity
Learning Task 3.3.1: Matching Mania
Instructions: Match the statements with the corresponding diagrams. A solid circle
indicates inclusion.

59
Prepared by: Rennil S. Bornia MST-Math
Self- Instructional Module on College and Advance Algebra & Advanced Mathematics

B.Analysis
Learning Task # 3.3.2: Writing Notations
Instructions: From the given statements above, write each statement in set-builder
notation and interval notation.
Statement Set-builder Notation Interval Notation
1. 𝑥 < −2, 𝑥 > 2
2. 𝑥 ≥ −2
3. 𝑥 > −2
4. 𝑥 ≤ 2
5. 𝑥 < 2
6. 𝑥 ≤ −2, 𝑥 > 2
7. 𝑥 ≥ 2
8. 𝑥 < −2
9. −2 ≤ 𝑥 < 2
10. 𝑥 > 2
11. 𝑥 ≤ −2

60
Prepared by: Rennil S. Bornia MST-Math
Self- Instructional Module on College and Advance Algebra & Advanced Mathematics

12. −2 < 𝑥 ≤ 2

B. Abstraction

Inequalities
An equation is a statement that indicates that two algebraic expressions are equal. An
inequality is a statement that indicates that two algebraic expressions are not equal in a
specific way, one expression being greater than or less than the other.

Inequality
Symbols
Symbol Meaning
< Is less than
≤ Is less than or equal to
> Is greater than
≥ Is greater than or equal to

• Inequality – is a statement that one algebraic expression is either less than or is less
than or equal to another algebraic expression.
• The domain of a variable in an inequality is the set of all real numbers for which both
sides of the inequality are defined.
• The real numbers that result in a true statement when those numbers are substituted for
the variable in the inequality are called solutions of the inequality.
• To solve an inequality means to find all solutions of the inequality – that is, its solution
set.

Linear Inequalities
• A linear inequality in one variable is an inequality that is equivalent to one of the forms
𝑎𝑥 + 𝑏 < 0 or 𝑎𝑥 + 𝑏 ≤ 0,
where 𝑎 and 𝑏 represent real numbers and 𝑎 ≠ 0.

61
Prepared by: Rennil S. Bornia MST-Math
Self- Instructional Module on College and Advance Algebra & Advanced Mathematics

RULES FOR INEQUALITIES


Rules Description
Addingthe same quantity to each side of an
1. 𝐴 ≤ 𝐵 𝐴+𝐶 ≤ 𝐵+𝐶
inequality gives an equivalent inequality.
Subtractingthe same quantity from each side of an
2. 𝐴 ≤ 𝐵 𝐴−𝐶 ≤ 𝐵 −𝐶
inequality gives an equivalent inequality.
Multiplyingeach side of an inequality by the same
3. If 𝐶 > 0, then 𝐴 ≤ 𝐵 𝐶𝐴 ≤ 𝐶𝐵
positivequantity gives an equivalent inequality.
Multiplyingeach side of an inequality by the same
4. If 𝐶 < 0, then 𝐴 ≤ 𝐵 𝐶𝐴 ≥ 𝐶𝐵 negativequantityreverses the direction
of an
inequality.
5. If 𝐴 > 0 and𝐵 > 0, Taking reciprocalsof each side of an inequality
1 1 involvingpositivequantities
reverses the direction
of
then 𝐴 ≤ 𝐵 ≥
𝐴 𝐵 the inequality
.
6. If 𝐴 ≤ 𝐵 and 𝐶 ≤ 𝐷, then 𝐴 + 𝐶 ≤ 𝐵 + 𝐷 Inequalities can be added.

Example 1: Solve each inequality and graph its solution set.


a. 7𝑥 − 11 < 2(𝑥 − 3) b. 8 − 3𝑥 ≤ 2
Solution:
a. 7𝑥 − 11 < 2(𝑥 − 3)
7𝑥 − 11 < 2𝑥 − 6 Distributive property
7𝑥 − 11 + 11 < 2𝑥 − 6 + 11 Add 11 to both sides
7𝑥 < 2𝑥 + 5 Simplify
7𝑥 − 2𝑥 < 2𝑥 + 5 − 2𝑥 Subtract 2x from both
sides
5𝑥 < 5 Simplify
Divide both sides by 5
𝑥<1 Simplify
The solution set is {𝑥|𝑥 < 1}, or in interval notation, (−∞, 1). The graph of the solution set is

b. 8 − 3𝑥 ≤ 2
8 − 3𝑥 − 8 ≤ 2 − 8 Subtract 8 from both sides
−3𝑥 ≤ −6 Simplify
Divide both sides by −3. (Reverse the direction of the inequality
symbol)
𝑥≥2 Simplify
The solution set is {𝑥|𝑥 ≥ 2}, or in interval notation, [2, ∞). The graph of the solution set is

62
Prepared by: Rennil S. Bornia MST-Math
Self- Instructional Module on College and Advance Algebra & Advanced Mathematics

Example 2. Solve the inequalities 4 ≤ 3𝑥 − 2 < 13.


Solution: The solution set consists of all values of 𝑥 that satisfy both of the inequalities 4
≤ 3𝑥 − 2 and 3𝑥 − 2 < 13. Using Rules 1 and 3, we see that the following
inequalities are equivalent.
4 ≤ 3𝑥 − 2 < 13
4 + 2 ≤ 3𝑥 − 2 + 2 < 13 + 2 Add 2
6 ≤ 3𝑥 < 15 Simplify
Divide by 3
2≤𝑥<5 Simplify
Therefore the solution set is {𝑥|2 ≤ 𝑥 < 5} or in interval notation
[2,5). The graph of the solution set is shown below.

¤ Solving Nonlinear Inequalities


To solve inequalities involving squares and other powers of the variable, we use
factoring, together with the following principle.

For example, to solve the inequality 𝑥2 − 5𝑥 ≤ −6, we first move we first move all terms to
the left hand side and factor to get (𝑥 − 2)(𝑥 − 3) ≤ 0.
This form of the inequality says that the product (𝑥 − 2)(𝑥 − 3) must be negative or zero, so
to solve the inequality, we must determine where each factor is negative or positive
(because the sign of a product depends on the sign of the factors). The details are
explained in Example 3, in which we use the following guidelines.

63
Prepared by: Rennil S. Bornia MST-Math
Self- Instructional Module on College and Advance Algebra & Advanced Mathematics

Example 3. Solving a Quadratic Inequality


Solve the inequality 𝑥2 ≤ 5𝑥 − 6.
Solution: We will follow the guidelines in solving nonlinear
inequalities. Step 1. Move all terms to one side.
𝑥2 ≤ 5𝑥 − 6 Given
inequality
2
𝑥 − 5𝑥 + 6 ≤ 0 Step 2. Subtract 5x, add 6
Factor.
(𝑥 − 2)(𝑥 − 3) ≤ 0 Factor
Step 3. Find the intervals.
The factors of the left-hand side are 𝑥 − 2 and 𝑥 − 3. These factors are zero
when 𝑥 is 2 and 3, respectively. As shown in the
figure on the right,
the numbers 2 and 3 divide the real line into the
three intervals
(−∞, 2), (2, 3), (3, ∞)
The factors 𝑥 − 2 and 𝑥 − 3 change sign only at 2 and 3, respectively. So these factors
maintain their sign on each of these three intervals.
Step 4. Make a table or diagram.
To determine the sign of each factor on each of the
intervals that we found, we use test values. We choose
a number inside each interval and check the sign of the
factors 𝑥 − 2 and 𝑥 − 3 at the number we chose. For
the interval (−∞, 2), let’s choose the test value 1 (see
figure on the right). Substituting 1 for 𝑥 in the factors 𝑥 − 2 and 𝑥 − 3, we get
𝑥 − 2 = 1 − 2 = −1 < 0
𝑥 − 3 = 1 − 3 = −2 < 0

64
Prepared by: Rennil S. Bornia MST-Math
Self- Instructional Module on College and Advance Algebra & Advanced Mathematics

So both factors are negative on this interval. Notice that we need to check only one test
value for each interval because the factors 𝑥 − 2 and 𝑥 − 3 do not change sign on any of
the three intervals we found.
Using the test values and 𝑥 = 4 for the intervals (2, 3), (3, ∞) respectively, we construct
the
following sign table. The final row of the table is obtained from the fact that the expression
in the last row is the product of the two factors.

If you prefer, you can represent this information on a real line, as in the following sign
diagram. The vertical lines indicate the points at which the real line is divided into intervals:

Step 5. Solve.
We read from the table or the diagram that (𝑥 − 2)(𝑥 − 3) is negative on the interval
(2, 3). Thus the solution of the inequality (𝑥 − 2)(𝑥 − 3) ≤ 0 is {𝑥|2 ≤ 𝑥 ≤ 3} or [2, 3]
We have included the endpoints 2 and 3 because we seek values of 𝑥 such that
the product is either less than or equal to zero. The solution is illustrated in the figure
below.

Example 4. Solving an Inequality with Repeated


Factors Solve the inequality 𝑥(𝑥 − 1)2(𝑥 − 3) < 0.
Solution:
All nonzero terms are already on one side of the inequality, and the nonzero side of
the inequality is already factored. So we begin by finding the intervals for this inequality.
Step 3. Find the intervals.
The factors of the left-hand side are . These are zero when
𝑥 = 0, 1, 3. These numbers divide the real line into the intervals
(−∞, 0), (0, 1), (1, 3), (3, ∞)
Step 4. Make a diagram.
We make the following diagram, using test points to determine the sign of each
factor in each interval:

65
Prepared by: Rennil S. Bornia MST-Math
Self- Instructional Module on College and Advance Algebra & Advanced Mathematics

Step 5. Solve.
From the diagram we see that 𝑥(𝑥 − 1)2(𝑥 − 3) < 0 for 𝑥 in the interval (0, 1) or for 𝑥 in
(1, 3). So the solution set is the union of these two intervals:
(0, 1) ∪ (1, 3)
The solution set is graphed below.

Example 5. Solving an Inequality Involving a Quotient


Solve the inequality .
Solution:
Step 1. Move all terms to one side.
Given inequality
Subtract 1
Common denominator 1 – 𝑥
Combine the fractions
Simplify

Step 3. Find the intervals. (Note: Proceed to step 3 since, the inequality is non-
factorable) The factors of the left-hand side are 2𝑥 and 1 − 𝑥. These are zero
when 𝑥 is 0 and 1. These numbers divide the real line into the intervals
(−∞, 0), (0, 1), (1, ∞)
Step 4. Make a diagram.
We make the following diagram using test points to determine the sign of each
factor in each interval:

Step 5. Solve.
From the diagram we see that in the interval [0,1). We include the endpoint

66
Prepared by: Rennil S. Bornia MST-Math
Self- Instructional Module on College and Advance Algebra & Advanced Mathematics

0 because the original inequality requires that the quotient be greater than or
equal to 1. However, we do not include the other endpoint 1 because the
quotient in the inequality is not defined at 1. So the solution set is {𝑥|0 ≤ 𝑥 < 1} or in
interval notation [0, 1). The solution set is graphed below.

D. Applications
Learning Task 3.3.3: Think-Pair-Solve
Identify the variable and write an inequality that describes each.
1. Billy is taller than 7 feet.
2. Jamaica is under 55 years old.
3. Jenny makes less than ₱800,000 per year.
4. Bob weighs over 80 pounds.
5. The maximum speed for the Concorde is 1450 miles per hour (mph).
6. The minimum speed on the freeway is 45 mph.
7. Julia can afford at most ₱20,000 per month.
8. Carl must have at least a 3.2 grade point average.
9. Brent is no taller than 6 feet.
10. James cannot run faster than 11 mph.
11. Kim makes no more than ₱410 per hour.
12. Rica will not take less than ₱600,000 for the car.
Solve each problem by using an inequality.
13. A 100-foot rope has to be cut in three pieces. The second piece is to be three times
as long as the first piece, and the third piece must be 18 feet long. What is the maximum
length of the second piece?
14. The width of a rectangle is 3 inches less than the length. The perimeter of the
rectangle can be no more than 50 inches. What is the maximum width?
15. The freshmen class is planning a picnic. The cost of a permit to use a city park is ₱12,500.
To pay for the
permit, there is a fee of ₱37.50 for each freshman and ₱62.50 for each guest who is not a
freshman. Two hundred freshmen plan to attend. Write and solve an inequality to find how
many guests must attend for the freshmen to pay for the permit.

IV. Assessment

Learning Task 3.3.4: Evaluation Quiz


Solve the linear inequality. Express the solution using interval notation and graph the
solution set.
1. 3𝑥 + 11 ≤ 6𝑥 + 8 2. 2𝑥 − 5 > 3 3. 6 − 𝑥 ≥ 2𝑥 + 9 4. 3𝑥 + 11 < 5 5. −2 < 8 − 2𝑥 ≤
−1

67
Prepared by: Rennil S. Bornia MST-Math
Self- Instructional Module on College and Advance Algebra & Advanced Mathematics

Solve the nonlinear inequality. Express the solution using interval notation and graph the
solution set.
1. (𝑥 − 5)(𝑥 + 4) ≥ 0

V. Enhancement
Learning Task 3.3.5: Graph Linear Inequalities in Two
Variables Instructions:
Watch YouTube videos on how to graph linear inequalities in
two variables. Link: https://www.youtube.com/watch?
v=5h6YzRRxzO4 Use graphing paper to graph the following
inequalities:
1. 𝑦 > 2𝑥 − 5
2. 3𝑥 − 2𝑦 < 10 5. 𝑦 ≥ −3𝑥 + 4
3. 5𝑥 − 3𝑦 ≤ −15

VI. References
Ratti, J. M., & Marcus, S. (2014). College Algebra and Trigonometry (3rd Ed.). Pearson
Education Limited.Stewart, J., Redlin, L., & Watson, S. (2015). Algebra and trigonometry.
Cengage Learning.
https://www.youtube.com/watch?v=5h6YzRRxzO4

UNIT 4: FUNCTIONS AND RELATIONS, LINEAR


AND
68
Prepared by: Rennil S. Bornia MST-Math
Self- Instructional Module on College and Advance Algebra & Advanced Mathematics

QUADRATIC FUNCTIONS
Unit Overview
. A function is a mathematical relationship
between two variables, where every input variable
has one output variable. The applications of
functions are everywhere in real life. For example,
the relationship between distance and time. We all
know that it takes time to travel distances and
when we travel any distance (or stand still), it takes
a certain amount of time to do so. The relationship
between distance and time is a functional
relationship. This unit discusses in detail the
several types of functions such as the piece-wise,
linear, and quadratic functions. This also tackles
how to find the equation and sketch it graph. The
definitions of related terms such as relation and
function, domain and range, linear and quadratic
functions are well stated to help you understand
clearly the topics in this unit. Various examples
with a step by step solution are also provided especially in sketching the graphs of different
types of functions.

Hence, this unit introduces functions, their notations, and their graphs. You will learn to
evaluate, graph, analyze, and create various types of functions. Lesson 4.1 gives you the
opportunity to evaluate functions, determine the domain of different types of functions, and
graph the basic types of functions. Lesson 4.2 allows you to find the equation of a line and
sketch a graph. Lastly, Lesson 4.3 provides you with opportunities to find the standard form
of quadratic function and graph.

Moreover, at the end of this unit, you should be able to:


 identify functions from relations;
 evaluate basic functions and piecewise functions;
 determine the domain and range of a function algebraically and graphically;
 graph basic type of functions and piecewise functions by plotting points;
 find the equation of a line and a standard form of quadratic function;
 graph linear function by finding the slope and y-intercept of each line;
 find the maximum or minimum value of the quadratic function;
 graph quadratic functions using transformations; and
 solve problems involving linear and quadratic functions;

69
Prepared by: Rennil S. Bornia MST-Math
Self- Instructional Module on College and Advance Algebra & Advanced Mathematics

Lesson 4.1: Functions and Relations


I. Desired Learning Outcomes

At the end of the lesson, you will be able to:


• Identify functions from relations;
• evaluate basic functions and piecewise functions;
• determine the domain and range of a function algebraically and graphically; 
graph basic type of functions and piecewise functions by plotting points; and 
solve problems involving functions.

II. Lesson Overview


The concept of function is one of the most important in mathematics. In this lesson, you
will start recalling what a function is by identifying functions from the given relations. This
section introduces function notation, types of functions, domain and range of functions.
Moreover, you will learn to evaluate, graph various types of functions.

III. Course Contents and Learning Experiences

A. Activity
Learning Task 4.1.1: Is It A Function?
Instructions: Determine which of the following relations below are functions by writing
YES if it is a function and NO if not.

1) {(−2, 7), (−1, 5), (0, 3), (1, 1), (2, 1)} 2) {(4, 8), (−3, −2), (2, −1), (−4, −5), (2, 7), }
3) 4)
𝑥 −2 −2 0 2 2 𝑥 −5 −3 −1 1 3 5
𝑦 −7 5 −16 0 6 𝑦 8 8 −2 −2 11 23

5) 6)

7) 8)

70
Prepared by: Rennil S. Bornia MST-Math
Self- Instructional Module on College and Advance Algebra & Advanced Mathematics

9) 𝑦 = −|𝑥 − 3| + 2 10) 𝑥 = −(𝑦 − 2)2 + 1


B. Analysis
Learning Task # 4.1.2: Think and Discuss
Instructions: Answer the following questions.
1. What do you think is a function?
2. Differentiate relation and function.
3. How did you identify that the set of ordered pairs is a function or not?
4. How did you identify that a table is a function or not?
5. How did you identify that a diagram is a function or not?
6. How did you identify that a graph is a function or not?
7. How did you identify that an equation is a function or not?

C.Abstraction
Key Concepts
• A relation is any set of ordered pairs.
• The set of all first components of the ordered pairs is called the domain of the
relation  The set of all second components is called the range.
• A relation in which each member of the domain corresponds to exactly one member
of the range is called a function.

Example 1: Given the set of points, determine whether the relation is a function or not.
{(1,5), (2,5),(2,8)} Answer: (2,5) and (2,8) have the same first components but different
second components. Thus, the relation is not a function.
Example 2: Given the equations, determine if the following relations define 𝑦 as a function
of 𝑥:
(a) 2𝑥 + 𝑦 = 3 (b) −𝑥2 + 𝑦2 = 16
Answer: (a) Solving 𝑦 in terms of 𝑥 in 2𝑥 + 𝑦 = 3 results to 𝑦 = −2𝑥 + 3. Since −2𝑥 + 3 is a
unique real number for each 𝑥, this equation defines 𝑦 as a function of 𝑥.
(b) Solving 𝑦 in terms of 𝑥 in −𝑥2 + 𝑦2 = 16 results to This implies
that the relation produces two values of 𝑦 for each 𝑥, positive value and
negative value. For example, when 𝑥 = 0, 𝑦 = 4, or 𝑦 = −4. Thus, −𝑥2 + 𝑦2 =
16 does not define 𝑦 as a function of 𝑥.
Note that if an equation is solved for 𝑦 and two or more values of 𝑦 are obtained for a given
x, then the equation does not define a function.

Example 3: Given the figure and the graph, determine if the relation defines 𝑦 as a function
of 𝑥.
(a) (b) (c)

71
Prepared by: Rennil S. Bornia MST-Math
Self- Instructional Module on College and Advance Algebra & Advanced Mathematics

Answer: (a) In the figure, each 𝑥 is paired with exactly one 𝑦. Thus, the correspondence
defines 𝑦 as a function of 𝑥. Notice that more than one element in the domain correspond
to the same element in the range.
(b) Since no vertical line intersects the graph in more than
one point, the graph defines 𝑦 as a function of 𝑥.
(c) The vertical line intersects the graph in more than one
point, thus, the graph does not define a function.
• Vertical Line Test – is used to determine whether the graph represents a function.
A graph in the 𝑥𝑦plane is a function if and only if no vertical line intersects the graph
in more than one point.

DEFINITION OF A FUNCTION
A function 𝑓 is a relation define as a set of ordered pairs (𝑥, 𝑦)
where no two or more distinct ordered pairs have the same first element
𝑥.
Every value of 𝑥 corresponds to a unique value of 𝑦.
¤ Function Notation
• The symbol 𝑓(𝑥) is read “𝑓 of 𝑥” or “𝑓 at 𝑥” and is called the value of 𝒇 at 𝒙.
• The set 𝐴 is called the domain of the function.
• The range of 𝑓 is the set of all possible values of 𝑓(𝑥) as x varies throughout the
domain, that is range of 𝑓 = {𝑓(𝑥)|𝑥 ∈ 𝐴}
• The symbol that represents an arbitrary number in the domain of a function 𝑓 is
called an independent variable.
• The symbol that represents a number in the range of 𝑓 is called a dependent
variable.
So, if we write 𝑦 = 𝑓(𝑥), then 𝑥 is the independent variable and 𝑦 is the dependent variable.
• Domain – is the set of all possible inputs
• Range is the set of all possible outputs

¤ Evaluation of Functions
Evaluating functions means finding the value of 𝑓(𝑥). To evaluate 𝑓(𝑥) at 𝑥 = 𝑎, simply
substitute 𝑎 for 𝑥 then simplify.
Example 4. Let 𝑓(𝑥) = 3𝑥2 + 𝑥 − 5. Evaluate each function value.
(a) 𝑓(−2) (b) 𝑓(0) (c) 𝑓(4)
Solution. To evaluate 𝑓 at a number, we substitute the number for 𝑥 in the definition of 𝑓.
(a) 𝑓(−2) = 3 ∙ (−2)2 + (−2) − 5 = 3 ∙ 4 − 2 − 5 = 12 − 2 − 5 = 5
(b) 𝑓(0) = 3 ∙ 02 + 0 − 5 = 0 + 0 − 5 = −5
(c) 𝑓(4) = 3 ∙ (4)2 + 4 − 5 = 3 ∙ 16 + 4 − 5 = 48 + 4 − 5 = 47

72
Prepared by: Rennil S. Bornia MST-Math
Self- Instructional Module on College and Advance Algebra & Advanced Mathematics

Example 5: If 𝑓(𝑥) = 2𝑥2 + 3𝑥 − 1, evaluate the following.


(a) 𝑓(𝑎) (b) 𝑓(−𝑎) (c) 𝑓(𝑎 + ℎ)
Solution
(a) 𝑓(𝑎) = 2𝑎2 + 3𝑎 − 1
(b) 𝑓(−𝑎) = 2(−𝑎)2 + 3(−𝑎) − 1 = 2𝑎2 − 3𝑎 − 1
(c) 𝑓(𝑎 + ℎ) = 2(𝑎 + ℎ)2 + 3(𝑎 + ℎ) − 1
= 2(𝑎2 + 2𝑎ℎ + ℎ2) + 3𝑎 + 3ℎ − 1
= 2𝑎2 + 4𝑎ℎ + 2ℎ2 + 3𝑎 + 3ℎ − 1
(d) Using the results from parts (c) and (a), we have
2 2 2

¤ The Domain of a Function


Domain – is the set of all possible x-values of the function.
When finding the domain of the function, take note of the following restrictions:
• The denominator of a fraction or rational function cannot be zero.
• The radicand or the number under a square root sign must be positive.
If the function has no restrictions as specified above, then the domain of the function is the
set of all real numbers.

Example 6: Find the domain of each function.

Solution.
(a)A rational expression is not defined when the denominator is 0. Since
, we
see that 𝑓(𝑥) is not defined when 𝑥 = 0 or 𝑥 = 1. Thus, the domain of 𝑓 is {𝑥|𝑥 ≠ 0, 𝑥 ≠ 1}.
The domain may also be written in interval notation as (−∞, 0) ∪ (0,1) ∪ (1, ∞).
(b)We can’t take the square root of a negative number, so we must have 9 − 𝑥2 ≥ 0.
We can solve this inequality to find that −3 ≤ 𝑥 ≤ 3. Thus, the domain of 𝑔 is {𝑥|
−3 ≤ 𝑥 ≤ 3} = [−3,3].
(c)We can’t take the square root of a negative number, and we can’t divide by 0, so
we must have 𝑡 +
𝑡 + 1 > 0, that is, 𝑡 > −1. So the domain of ℎ is {𝑡|𝑡 > −1} = (−1, ∞)

¤ Graph of a Function

73
Prepared by: Rennil S. Bornia MST-Math
Self- Instructional Module on College and Advance Algebra & Advanced Mathematics

The graph of a function 𝑓 on the 𝑥𝑦-plane consists of points (𝑥, 𝑦) such that 𝑥 is in the
domain of 𝑓 and 𝑦 = 𝑓(𝑥). To graph a function through a table of values, we can follow
these steps.
1. Select at least five values of 𝑥 in the domain. The choice of x-values is arbitrary.
2. Find the corresponding values of the function for each selected 𝑥.
3. Plot the obtained ordered pairs or points on the coordinate plane then, join them with
a smooth curve.

Example 7: Graph each of the following. State the domain of each function.
a. 𝑓(𝑥) = 2𝑥 − 3 b. 𝑔(𝑥) = 2𝑥2 − 3 c.
Solution:
a. Because 2𝑥 − 3 is a real number for all values ofx, the domain of
𝑓 is all real numbers. This can be written( −∞, ∞ ) .

Plot each ordered pair, and then draw a smooth graph through the points. The
graph is shown on the right.

b. Because 2𝑥2 − 3 is a real number for all values of x,


the domain of
𝑔 is all real
numbers. This can be written (−∞, ∞).

Plot each ordered pair, and then draw a smooth graph through the points. The graph is
shown on the right.

c. Because 2 √ 𝑥 − 3 is not a real number when𝑥 < 0 , the


domain ofℎ is all real numbers greater than or equal to zero. This can
be written[0, ∞).

Plot each ordered pair, and then draw a smooth graph through the
points. The graph is shown on the right.

¤ Piecewise Functions
Piecewise functions are functions defined by more than one sub-functions where each
sub-function has a particular domain.

74
Prepared by: Rennil S. Bornia MST-Math
Self- Instructional Module on College and Advance Algebra & Advanced Mathematics

Example 7: Evaluate
(a) 𝑓(−2) (b) 𝑓(0) (c) 𝑓(3)
Solution:
(a)Evaluate 𝑓(𝑥) = −𝑥2 + 11 since −2 < 0.
𝑓(−2) = −𝑥2 + 11 = −(−2)2 + 11 = −4 + 11 = 7
(b)Evaluate 𝑓(𝑥) = −𝑥2 + 11 since 0 = 0.
𝑓(0) = −02 + 11 = 11
(c)Evaluate 𝑓(𝑥) = 3𝑥 + 4 since 3 > 0
𝑓(3) = 3𝑥 + 4 = 3(3) + 4 = 9 + 4 = 13

¤ Graph of a Piecewise Function


To sketch the graph of a piecewise function, here are the following steps:
1. Make a table of values for each piece of the function.
2. Plot the obtained points on the same coordinate plane, then connect the points for
each piece.
3. In the given interval for each piece, check the graphs if they satisfy the vertical line
test.

D.Applications
Learning Task 4.1.3: Think-Pair-Solve
Instructions: Solve the following problems. Show the illustration if necessary.
1. The volume 𝑉 of a right circular cone is . If the height is twice the radius,
express the volume 𝑉 as a function of radius 𝑟.
2. A wire measuring 𝑥 is bent into a square shape. Do the following:
a. Express the perimeter 𝑃 of the square as a function of 𝑥.
b. Express the area 𝐴 of the square as a function of 𝑥.
3. An open box with a square base is required to have a volume of 10𝑓𝑡3.
a. Express the amount 𝐴 of the material to be use, to make the required box, as a
function of the length 𝑥 of a side of the square base.
b. Find the amount of the material required for a base of 1 𝑓𝑡 by 1 𝑓𝑡.
4. At exactly 1: 00 p.m., a hot-air balloon was released and rose vertically at a rate of 2
𝑚/𝑠. An observer was 100 𝑚 away from a point on the ground directly below the
balloon. Express the distance 𝑑 between the balloon and the observer as a function of 𝑡
where 𝑡 refers to the time in seconds after 1: 00 p.m.

75
Prepared by: Rennil S. Bornia MST-Math
Self- Instructional Module on College and Advance Algebra & Advanced Mathematics

5. At 6: 00 a.m., two ships leave a port. One ship is sailing south at a rate of 18 𝑘𝑝ℎ and
the other west at a rate of 23 𝑘𝑝ℎ. Express the distance 𝑑 between the two ships as a
function of 𝑡 where 𝑡 denotes the time in ℎ𝑟𝑠 after 7: 00 a.m.

IV. Assessment
Learning Task 4.1.4: Evaluation Quiz
Find the domain of the function algebraically. Write your answer in set-builder notation.
1.
√ 2.

3.

4.

5.

Evaluate the function at the indicated values.


11. 𝑓(𝑥) = 𝑥3 + 2𝑥;

a. 𝑓(−2) b. 𝑓(1) c.
;
a. 𝑔(−2) b. c. 𝑔(𝑎 − 1)
13. 𝑓(𝑥) = 6𝑥 − 18

a. b. 𝑓(𝑥 + 2) c. 𝑓(𝑥2 + 5)

a. 𝑓(−3) b. 𝑓(0) c. 𝑓(2)


3𝑥 𝑖𝑓 𝑥 < 0
15. 𝑓(𝑥) = {𝑥 + 1 𝑖𝑓 0 ≤ 𝑥 ≤ 2
(𝑥 − 2)2 𝑖𝑓 𝑥 > 2
a. 𝑓(−5) b. 𝑓(0) c. 𝑓(1) d. 𝑓(2) e. 𝑓(5)
Use a graphing paper to graph each function by plotting at least 5 points.
16. 𝑥2 − 4 17. 2𝑥 − 4

−𝑥 𝑖𝑓 𝑥 ≤ 0

76
Prepared by: Rennil S. Bornia MST-Math
Self- Instructional Module on College and Advance Algebra & Advanced Mathematics

V. Enhancement

Learning Task 4.1.5: Video Analysis


Instructions: Watch the YouTube Video on how to find the domain and range of the
function graphically.
Link: https://www.youtube.com/watch?v=m31DL444Yx0
Then, find the domain and range of the following graphs of functions. Write your answer in
interval notation.
1. 2. 3.

4. 5. 6.

VI. References

Ratti, J. M., & Marcus, S. (2014). College Algebra and Trigonometry (3rd Ed.). Pearson
Education Limited.
Stewart, J., Redlin, L., & Watson, S. (2015). Algebra and trigonometry. Cengage Learning.
Flores, M., Gagani, R.F., & Ypanto, Q. (2016). Worktext in General Mathematics: Activity-
based, Scaffolding of Student Learning Approach for Senior High School. C & E.

77
Prepared by: Rennil S. Bornia MST-Math
Self- Instructional Module on College and Advance Algebra & Advanced Mathematics

LESSON 4.2: LINEAR FUNCTIONS


I. Desired Learning Outcomes

At the end of the lesson, you will be able to:


• write the function rule of the linear relationship;
• find the slope of the line;
• graph linear function by finding the slope and y-intercept of each line;
• find the equation of a line; and
• solve problems involving linear functions.

II. Lesson Overview

In the previous lesson, you learned about the concept of functions and its characteristics.
In this lesson, you will learn how to write function rule based on the table of values. You will
also learn how to write an equation of a linear function when given a set of data. You will
also be given an opportunity to graph a linear function with the given slope and y-intercept.
Lastly, you will learn some applications of linear functions in real-life.

III. Course Contents and Learning Experiences

A. Activity

Learning Task 4.2.1: What’s the Function Rule?


A. Write a function rule for the relationship in the table.
1. 2.

3. 4.

5. 6.

7. 8.

78
Prepared by: Rennil S. Bornia MST-Math
Self- Instructional Module on College and Advance Algebra & Advanced Mathematics

9. 10.

B. Identify
the

independent and dependent variables.


11. 12.

13. 14.

79
Prepared by: Rennil S. Bornia MST-Math
Self- Instructional Module on College and Advance Algebra & Advanced Mathematics

15. 16.

17. 18.

B. Analysis
Learning Task 4.2.2: Think and Discuss Activity
1. Describe the relationship that you have observed in the table.
2. What would be the possible graph in numbers 1 to 10?
3. What is the type of function described in the activity?

C. Abstraction

¤ Linear Functions

The domain of a linear function is the interval (−∞, ∞) because the expression 𝑚𝑥 + 𝑏 is
defined for any real number. The range of a non-constant linear function also is the interval
(−∞, ∞) because the graph extends indefinitely upwards and downward. The range of a
constant function 𝑓(𝑥) = 𝑏 is the single real number 𝑏.

¤ Slopes of Lines
• A linear function is a function that can be written in the form 𝑓(𝑥) = 𝑚𝑥 + 𝑏. Its graph
is a straight line.

80
Prepared by: Rennil S. Bornia MST-Math
Self- Instructional Module on College and Advance Algebra & Advanced Mathematics

• The slope of a line is a constant rise or fall of a graph of a linear function.


• The graph of a line has a positive slope when the y-value increases as x increases
at the same time. On the other hand, the graph of a line has a negative slope when
the y-value decreases as x increases.

Example 1. Find the slope of the line passing through 𝑃1 and 𝑃2.
a. 𝑃1(1, 2), 𝑃2(3, 6) b. 𝑃1(−3, 4), 𝑃2(1, −2)
Solution:
a.
Because 𝑚 > 0, the line slants upward from left to right. The slope of the line is positive.

b.
Because 𝑚 < 0, the line slants downward from left to right. The slope of the line is
negative.

¤ Graph of a Linear Function


• If a function is written in the form 𝑓(𝑥) = 𝑚𝑥 + 𝑏 then its graph can be drawn by first
plotting the yintercept and then using the slope to determine another point on the line.

81
Prepared by: Rennil S. Bornia MST-Math
Self- Instructional Module on College and Advance Algebra & Advanced Mathematics

Example 2. Graph 𝑓(𝑥) = 2𝑥 − 1


Solution: Replace 𝑓(𝑥) with 𝑦. The equation 𝑦 = 2𝑥 − 1 is in
slope-intercept form, with 𝑏 = −1 and 𝑚 = 2. Thus the y-
intercept is (0, −1) and the slope is 2. Write the slope as

.
To graph the equation, first plot the 𝑦- intercept and then use the
slope to plot a second point. This second point is 2 units up
(change in y) and 1 unit to the right (change in x) of the y-
intercept.

• An equation of the form 𝐴𝑥 + 𝐵𝑦 = 𝐶, where 𝐴, 𝐵, and 𝐶 are real numbers and both 𝐴
and 𝐵 are not zero, is called general form of a linear equation in two variables.

• To graph a linear equation in general form, you can solve for 𝑦 and then proceed as
previously shown in Example 2.

Example 3: Graph 3𝑥 + 2𝑦 = 4
Solution: Solve for y. 3𝑥 + 2𝑦 = 4
2𝑦 = 4 − 3𝑥

The slope is , and the y-intercept is (0, 2).


82
Prepared by: Rennil S. Bornia MST-Math
Self- Instructional Module on College and Advance Algebra & Advanced Mathematics

To graph the equation, plot the y-intercept and then use the slope to plot a second
point. The second point is 3 units down (change in y) and 2 units to the right (change in
x) from the y-intercept.

• The graph in Example 3 can also be drawn by first finding the 𝑥 − and 𝑦 −intercepts
and then drawing a straight line through those points.

To find the 𝑥-intercept, let 𝑦 To find the 𝑦-intercept, let 𝑥 =


= 0 and solve for 𝑥. 0 and solve for 𝑦.
3𝑥 + 2𝑦 = 4 3𝑥 + 2𝑦 = 4
3𝑥 + 2(0) = 4 3(0) + 2𝑦 = 4
3𝑥 = 4 2𝑦 = 4
𝑦=2
The 𝑦-intercept is (0, 2).
The 𝑥-intercept is .
Plot the 𝑥 − and 𝑦 −intercepts and then draw a line through
the points.

¤ Finding the Equation of a Line


• We can find an equation of a line provided we know its slope and at least one point on
the line.

Example 4. Find an equation of the line with slope −3 that passes through (−1, 4).
Solution: Use the point-slope form with 𝑚 = −3, 𝑥1 = −1, and 𝑦1 = 4.
𝑦 − 𝑦1 = 𝑚(𝑥 − 𝑥1)
𝑦 − 4 = −3[𝑥 − (−1)] Substitute
𝑦 − 4 = −3𝑥 − 3 Solve for 𝑦
𝑦 = −3𝑥 + 1 Slope-intercept form
Example 5. Find the equation of a line that passes through 𝑃1(−2, 4) and 𝑃2(2, −1).
Solution: To find the equation of a line through two given points, first find the slope of the line.

The slope is . Now use the point-slope form.


𝑦 − 𝑦1 = 𝑚(𝑥 − 𝑥1) Use the point-slope form.
Substitute

83
Prepared by: Rennil S. Bornia MST-Math
Self- Instructional Module on College and Advance Algebra & Advanced Mathematics

Solve for 𝑦
Simplify and write in slope-intercept form.

¤ Parallel and Perpendicular Lines


• Two nonintersecting lines in a plane are parallel.
• Two lines are perpendicular if and only if they intersect and form adjacent angles,
each of which measures 90°.

Example 6: Find Equations of Parallel and Perpendicular Lines


a.Find the equation of the line whose graph is parallel to the graph of 2𝑥 − 3𝑦 = 7 and
passes through the point 𝑃(−6, −2).
b.Find the equation of the line whose graph is perpendicular to the graph of and
passes through the point 𝑃(−4, 1). Solution:
a.Solving 2𝑥 − 3𝑦 = 7 for 𝑦, we have . Therefore, the slope of a line parallel
to the given lines is . Now use the point-slope form with .
𝑦 − 𝑦1 = 𝑚(𝑥 − 𝑥1) Use the point-slope form.
Substitute
Solve for 𝑦
Simplify and write in slope-intercept form.
The equation of the line whose graph is parallel to the graph of 2𝑥 − 3𝑦 = 7 and
passes
through the point

84
Prepared by: Rennil S. Bornia MST-Math
Self- Instructional Module on College and Advance Algebra & Advanced Mathematics

b.The slope of the given line is . The slope of a line perpendicular to the given line is
the negative reciprocal of . Now use the point–slope form with and
𝑃(−4, 1).
𝑦 − 𝑦1 = 𝑚(𝑥 − 𝑥1) Use the point-slope form.
Substitute
Solve for 𝑦
Simplify and write in slope-intercept form.
The equation of the line whose graph is perpendicular to the graph of
and passes through the point .

D. Application
Learning Task 4.2.3: Think-Pair-Solve

A. Students in the Science class are selling candles to raise money for an upcoming trip. Each
student has 24 candles to sell. If a student sells 4 candles a profit of ₱1,500 is made. If he sells 12
candles a profit of ₱3,500 is made. The profit and the number of candles sold form a linear relation.

1. What is the dependent and independent variables?


2. Make a table of values for this relationship.
3. Draw the graph and write the label for x-axis and y-axis.
4. Write an equation which represents this situation.
5. Find the slope and explain what it means in this problem.
6. Find the profit-intercept and explain what it represents.
7. Solve for the maximum profit that a student can make.
8. Find a suitable domain and range.
9. If a student makes a profit of ₱4,500 , how many candles did he sell?
10. Is this data continuous, discrete, or neither? Justify your answer.

B. Mark leaves his home and drives to school. After driving for 5 hours, he is 112 km from school,
and after 7 hours, he is 15 km from school. Assume that the distance from school and the number
of hours driving form a linear relationship.
11. What is the dependent and independent variables?
12. Make a table of values for this relationship.
13. Draw the graph and write the label for x-axis and y-axis.
14. Write an equation which represents this situation.
15. Find the slope and explain what it means in this problem.
16. Find the distance-intercept and explain its real-life meaning in this problem.
17. How long did it take Mark to drive from his home to school?
18. Find a suitable domain and range.
19. How far was Mark from school after driving 4 hours?
20. How long had Mark been driving when he was 209 km from school?

85
Prepared by: Rennil S. Bornia MST-Math
Self- Instructional Module on College and Advance Algebra & Advanced Mathematics

IV. Assessment
Learning Task 4.2.4: Evaluation Quiz

Find the slope of the line that passes through the given points.
1. (3, 4) and (1, 7) 2. (−2, 4) and (5, 1) 3. (−5, −1) and (−3, 4)

Graph 𝑦 as a function of 𝑥 by finding the slope and y-intercept of


each line.
3

6. 𝑦 = 2𝑥 − 4
9. 2𝑥 − 5𝑦 = −15 10. 4𝑥 + 3𝑦 − 12 = 0

Find the equation of the indicated line. Write the equation in the form 𝑦 = 𝑚𝑥 + 𝑏.

11. 𝑦-intercept: (0, 5); slope: −2 12. 𝑦-intercept: slope:


13. 𝑦-intercept: (0, −1); slope: 14. Through (−5, −1); slope: −3
15. Through (3, 1) and (−1, 4) 16. Through (−5, 6) and (−3, −4)

Find the equation of the line, in slope-intercept form, that satisfies the given conditions.
17. The graph is parallel to the graph of 𝑦 = 2𝑥 + 3 and passes through the point whose
coordinates are (2, −4).
18. The graph is parallel to the graph of and passes through the point whose
coordinates are (−4, 2).
19. The graph is parallel to the graph of 2𝑥 − 5𝑦 = 2 and passes through the point whose
coordinates are (5, 2).
20. The graph is perpendicular to the graph of 𝑦 = 2𝑥 − 5 and passes through the point whose
coordinates are (3, −4).
21. The graph is perpendicular to the graph of and passes through the point whose
coordinates are (6, 0).
22. The graph is perpendicular to the graph of −𝑥 − 4𝑦 = 6 and passes through the point whose
coordinates are (5, 2).

V. Enhancement

Learning Task 4.2.5: KWL Chart Reflection


Fill out the KWL Chart below with the information you have learned in this lesson.

86
Prepared by: Rennil S. Bornia MST-Math
Self- Instructional Module on College and Advance Algebra & Advanced Mathematics

VI. References
Ratti, J. M., & Marcus, S. (2014). College Algebra and Trigonometry (3rd Ed.). Pearson
Education Limited. Stewart, J., Redlin, L., & Watson, S. (2015). Algebra and trigonometry.
Cengage Learning.

LESSON 4.3: QUADRATIC FUNCTIONS


I. Desired Learning Outcomes

At the end of the lesson, you will be able to:


• find the standard form of the quadratic function by completing the square;
• use the vertex formula to determine the vertex of the quadratic function;
• find the range of quadratic function; and
• find the maximum or minimum value of the quadratic function.

II. Lesson Overview


Quadratic functions are used in many types of real world situations. They are useful in
describing the trajectory of a ball, determining the height of a thrown object and in
optimizing profit for businesses. In this lesson, you will start reviewing the process of
solving quadratic equation by completing the square. You will be given opportunities to find
standard form of the quadratic function by completing the square and use vertex formula.
Moreover, you will also learn to find the range of the quadratic functions, as well as its
minimum and the maximum value. At the end of this lesson, you will make a video
presentation on graphing quadratic functions using transformations.

III. Course Contents and Learning Experiences

A. Activity
Learning Task 4.3.1
Instructions: Solve the quadratic equations by completing the square.

87
Prepared by: Rennil S. Bornia MST-Math
Self- Instructional Module on College and Advance Algebra & Advanced Mathematics

1. 2.
3.
4.
5.
6.
7.

B. Analysis
Learning Task 4.3.2: Think and Discuss
Instructions: Answer the following
questions.
1.What is the graph of quadratic function?
2.What is the domain of quadratic function?
3.Describe the graph of quadratic function when
4.Describe the graph of quadratic function when

C. Abstraction

88
Prepared by: Rennil S. Bornia MST-Math
Self- Instructional Module on College and Advance Algebra & Advanced Mathematics

¤ Finding the Standard Form of a Quadratic Function


Example 1. Use the technique of completing the square to find the standard form of 𝑔(𝑥) =
2𝑥2 − 12𝑥 + 19.
Sketch the graph.
Solution:
𝑔(𝑥) = 2𝑥2 − 12𝑥 + 19
= 2(𝑥2 − 6𝑥) + 19 Factor 2 from the variable terms.
= 2(𝑥 − 6𝑥 + 9 − 9) + 19
2
Complete the square.
2
= 2(𝑥 − 6𝑥 + 9) − 2(9) + 19 Regroup.
= 2(𝑥 − 3)2 − 18 + 19 Factor and simplify.
= 2(𝑥 − 3)2 + 1 Use standard form.
The vertex is (3, 1). The axis of symmetry is 𝑥 = 3. Because 𝑎 > 0,
the parabola opens up.
¤ Vertex of a Parabola

• The vertex formula can be used to write the standard form of the equation of a
parabola.
• We have and

Example 2. Find the Vertex and Standard Form of a Quadratic Function


Use the vertex formula to find the vertex and standard form of 𝑓(𝑥) = 2𝑥2 − 8𝑥 + 3.
Solution: 𝑓(𝑥) = 2𝑥2 − 8𝑥 + 3 𝑎 = 2, 𝑏 = −8, 𝑐 = 3

89
Prepared by: Rennil S. Bornia MST-Math
Self- Instructional Module on College and Advance Algebra & Advanced Mathematics

x-coordinate of the vertex


y-coordinate of the vertex
The vertex is (2, −5). Substituting into the standard form equation 𝑓(𝑥) = 𝑎(𝑥
− ℎ)2 + 𝑘 yields the standard form 𝑓(𝑥) = 2(𝑥 − 2)2 − 5.

¤ Maximum and Minimum of a Quadratic Function

Based on the graph of 𝑓(𝑥) = 2𝑥2 − 8𝑥 + 3 as shown above, the parabola opens up and the
vertex is the lowest point on the graph of the parabola. Therefore, the y-coordinate of the
vertex is the minimum value of that function. This information can be used to determine the
range of 𝑓(𝑥) = 2𝑥2 − 8𝑥 + 3. The range is {𝑦|𝑦 ≥ −5}. Similarly, if the graph of a parabola
opened down, the vertex would be the highest point on the graph and the ycoordinate of
the vertex would be the maximum value of the function. For instance, the maximum value
of 𝑓(𝑥) = −𝑥2 + 4𝑥 − 1, graphed above, is 3, the y-coordinate of the vertex. The range of the
function is {𝑦|𝑦 ≤ 3}. For the two functions given above, the domain is the set of real
numbers.

¤ Find the Range of 𝒇(𝒙) = 𝒂𝒙𝟐 + 𝒃𝒙 + 𝒄


Example 3: Find the range of 𝑓(𝑥) = −2𝑥2 − 6𝑥 − 1. Determine the values of 𝑥 for which 𝑓(𝑥)
= 3. Solution: To find the range of 𝑓, determine the y-coordinate of the vertex of the graph of
𝑓.

𝑓(𝑥) = −2𝑥2 − 6𝑥 − 1 𝑎 = −2, 𝑏 = −6, 𝑐 = −1 x-

coordinate of the vertex y-coordinate of


the vertex
7
The vertex is . Because the parabola opens down, is the maximum value of 𝑓.
Therefore, the range of
2

is .
To determine the values of 𝑥 for which 𝑓(𝑥) = 3, replace 𝑓(𝑥) with −2𝑥2 − 6𝑥 − 1 and solve for 𝑥.
𝑓(𝑥) = 3
−2𝑥2 − 6𝑥 − 1 = 3 Replace 𝑓(𝑥) with −2𝑥2 − 6𝑥 − 1
−2𝑥2 − 6𝑥 − 4 = 0 Solve for 𝑥
90
Prepared by: Rennil S. Bornia MST-Math
Self- Instructional Module on College and Advance Algebra & Advanced Mathematics

−2(𝑥 + 1)(𝑥 + 2) = 0 Factor


𝑥+1=0 or 𝑥 + 2 = 0 Use the zero product principle to solve for 𝑥.
𝑥 = −1 𝑥 = −2
The values of 𝑥 for which 𝑓(𝑥) = 3 are −1 and −2.

¤ Find the Maximum or Minimum of a Quadratic Function


Example 4. Find the maximum or minimum value of each quadratic function. State whether
the value is a maximum or a minimum.
2
a. 𝑓(𝑥) = −2𝑥 + 8𝑥 − 1 b. 𝑔(𝑥) = 𝑥2 − 3𝑥 + 1
Solution: The maximum or minimum value of a quadratic function is the y-coordinate of
the vertex of the graph of the function.

a.
Because 𝑎 < 0, the function has a maximum value but no minimum value. The
maximum value is 7.

b.
Because 𝑎 > 0, the function has a minimum value but no maximum value. The minimum
value is .
D. Application
Learning Task 4.3.3: Think-Pair-Solve

91
Prepared by: Rennil S. Bornia MST-Math
Self- Instructional Module on College and Advance Algebra & Advanced Mathematics

1.
b.

Write a quadratic function that models each


graph. a.

92
Prepared by: Rennil S. Bornia MST-Math
Self- Instructional Module on College and Advance Algebra & Advanced Mathematics

IV. Assessment

Learning Task 4.3.4: Evaluation Quiz


A. Match each graph with the proper quadratic function.

𝒇(𝒙) = 𝒙𝟐 − 𝟑 𝒇(𝒙) = −𝟐𝒙𝟐 + 𝟐 𝒇(𝒙) = 𝒙𝟐 + 𝟐

. 𝒇(𝒙) = (𝒙 − 𝟒)𝟐 𝒇(𝒙) = (𝒙 + 𝟏)𝟐 + 𝟑 𝒇(𝒙) = (𝒙 + 𝟑)𝟐 𝒇(𝒙) = −𝟐(𝒙 − 𝟐)𝟐 + 𝟐


1. 2. 3.

4. 5. 6.

7. 8.

B. Use the method of completing the square to find the standard form of the quadratic
function. State the vertex and axis of symmetry of the graph of the function.
9. 𝒇(𝒙) = 𝒙𝟐𝟒𝒙 + 𝟏 10. 𝒇(𝒙) = 𝒙𝟐 + 𝟔𝒙 − 𝟏 11. 𝒇(𝒙) = −𝒙𝟐 + 𝟒𝒙 + 𝟐
12. 𝒇(𝒙) = −𝟑𝒙𝟐 + 𝟑𝒙 + 𝟕 13. 𝒇(𝒙) = −𝟐𝒙𝟐 − 𝟒𝒙 + 𝟓

93
Prepared by: Rennil S. Bornia MST-Math
Self- Instructional Module on College and Advance Algebra & Advanced Mathematics

C. Use the vertex formula to determine the vertex of the graph of the function and write the
function in standard form.
14. 𝒇(𝒙) = 𝒙𝟐 − 𝟏𝟎𝒙 15. 𝒇(𝒙) = 𝒙𝟐 − 𝟏𝟎 16. 𝒇(𝒙) = −𝒙𝟐 + 𝟔𝒙 + 𝟏
17. 𝒇(𝒙) = 𝟐𝒙𝟐 − 𝟑𝒙 + 𝟕 18. 𝒇(𝒙) = −𝟒𝒙𝟐 + 𝒙 + 𝟏

D. Find the range of the given quadratic function.


18. 𝒇(𝒙) = 𝒙𝟐 − 𝟐𝒙 − 𝟏 19. 𝒇(𝒙) = −𝒙𝟐 − 𝟔𝒙 − 𝟐 20. 𝒇(𝒙) = −𝟐𝒙𝟐 + 𝟓𝒙 − 𝟏
21. 𝒇(𝒙) = 𝟐𝒙𝟐 + 𝟔𝒙 − 𝟓 22. 𝒙𝟐 − 𝟔𝒙

E. Find the maximum or minimum value of the function. State whether this value is a
maximum or a minimum.
37.𝒇(𝒙) = 𝒙𝟐 + 𝟖𝒙 38. 𝒇(𝒙) = −𝒙𝟐 + 𝟔𝒙 + 𝟐
39. 𝒇(𝒙) = 𝟓𝒙𝟐 − 𝟏𝟏

V. Enhancement
Learning Task 4.3.5: Graphing Transformations
Make a 2-minute video on how to graph quadratic functions using transformations. You
have to show the process of graphing by giving examples.

VI. References

Ratti, J. M., & Marcus, S. (2014). College Algebra and Trigonometry (3rd Ed.). Pearson
Education Limited. Stewart, J., Redlin, L., & Watson, S. (2015). Algebra and trigonometry.
Cengage Learning.

94
Prepared by: Rennil S. Bornia MST-Math
Self- Instructional Module on College and Advance Algebra & Advanced Mathematics

UNIT 5: ALGEBRA OF FUNCTIONS, RATIONAL


FUNCTIONS
AND INVERSE FUNCTIONS
Unit Overview
The previous unit introduces the concepts of
functions and the basic types of functions and their
graphs. In this unit, we will study the basic
operations of functions which involves adding,
subtracting, multiplying, and dividing functions as
well to create a new function. We are also going to
include the new method of combining functions
which is the composition of functions. We apply
composite functions when we buy discounted
items that we do not realize that we can apply.
Moreover, other types of functions like rational and
inverse functions are also included.

Hence, this unit enables students to learn to


combine functions, determine the domain of
functions and its graphs. Lesson 5.1 you will learn
to perform and evaluate algebra of functions and
composite functions. Lesson 5.2 allows you to evaluate and graph rational functions.
Lastly, Lesson 4.3 provides you with opportunities to find the inverse of a function.

Moreover, at the end of this unit, you should


be able to:  define basic operations on
functions;
• evaluate algebra of function and composite functions;
95
Prepared by: Rennil S. Bornia MST-Math
Self- Instructional Module on College and Advance Algebra & Advanced Mathematics

• perform algebra of functions, composition, and decomposition of functions;


• determine the domain after performing algebra of functions and composite
functions;
• define the domain and asymptotes of rational functions;
• determine the domain of the rational functions
• find vertical and horizontal asymptotes; and
• transform graphs of rational functions
• determine the inverse of a function
• graph functions and their inverses
• solve problems involving composite functions, rational and inverse functions.

Lesson 5.1: Algebra of Functions


I. Desired Learning Outcomes

At the end of the lesson, you will be


able to:  define basic operations
on functions;
• evaluate algebra of function;
• perform algebra of functions;
• determine the domain after performing algebra of functions;
• evaluate composite function;
• perform composition and decomposition of functions;  determine the domain of
composite functions; and  solve problems involving composition of functions.

II. Lesson Overview


Knowing how a function is “put together” is an important first step when applying the tools
of calculus. Functions have their own algebra based on the same operations we apply to
real numbers (addition, subtraction, multiplication, and division). One way to build new
functions is to apply these operations. Thus, this lesson incorporates algebraic techniques
in performing algebra of functions especially in composition of functions and decomposition
of functions.

III. Course Contents and Learning Experiences

A. Activity
96
Prepared by: Rennil S. Bornia MST-Math
Self- Instructional Module on College and Advance Algebra & Advanced Mathematics

Learning Task 5.1.1: Let’s Investigate

1. Add:
2. Subtract: (2𝑥2 + 3𝑥 − 4) − (𝑥2 + 3𝑥 − 5)
3. Multiply: (3𝑥2 − 𝑥 + 2)(2𝑥 − 3)
2 3

4. Divide:
5. Let: 𝑓(𝑥) = 2𝑥2 − 5𝑥 + 2. Find the following
a. 𝑓(3𝑎) b. 𝑓(2 + ℎ)
6. Find the domain of each function.
a. b.

B. Analysis
Learning Task # 5.1.2: Think and Discuss
1.What are the basic operations with functions?
2.How do we perform the basic operations with functions?
3.What is the composition of two functions?
4.How is the order of operations related to composition of functions?

C. Abstraction

¤ Algebra of Functions
Functions are mostly defined by the four fundamental operations of algebra. For
instance, the function defined by ℎ(𝑥) = 𝑥2 + 5𝑥 may be regarded as the sum of 𝑓(𝑥)
= 𝑥2 and 𝑔(𝑥) = 5𝑥. This idea also works in the remaining operations. Thus, given any

two functions 𝑓 and 𝑔, we can define the four new functions .


ALGEBRA OF FUNCTIONS
Let 𝑓 and 𝑔 be functions with domains 𝐴 and 𝐵, respectively, then
The sum (𝑓 + 𝑔)(𝑥) = 𝑓(𝑥) + 𝑔(𝑥) Domain: 𝐴 ∩ 𝐵
The difference (𝑓 − 𝑔)(𝑥) = 𝑓(𝑥) − 𝑔(𝑥) Domain: 𝐴 ∩ 𝐵
The product (𝑓𝑔)(𝑥) = 𝑓(𝑥)𝑔(𝑥) Domain: 𝐴 ∩ 𝐵
The quotient 𝑓 𝑓(𝑥) Domain: 𝐴 ∩ 𝐵, 𝑔 ≠ 0
( )( ) = 𝑥
𝑔 𝑔(𝑥)

By the definition of the algebra of functions, domains of are the


intersection of the

domain of 𝑓 and 𝑔 – the x-values common to both domains. While, the domain of is
the subset of the intersection of domains of 𝑓 and 𝑔 consisting of all 𝑥 such that 𝑔 ≠ 0.

97
Prepared by: Rennil S. Bornia MST-Math
Self- Instructional Module on College and Advance Algebra & Advanced Mathematics

Example 1: Let .
Find the following functions and their domains.
a. 𝑓 + 𝑔 b. 𝑓 + ℎ c. 𝑔 − ℎ d. ℎ𝑔 e.
Solution:
a. . Domain: {𝑥|𝑥 ≥ 0 𝑎𝑛𝑑 𝑥 ≠ 2}
b.

Domain: {𝑥|𝑥 ∈ ℝ 𝑎𝑛𝑑 𝑥 ≠ 2}


c. . Domain: {𝑥| 𝑥 ≥ 0}
d. (ℎ𝑔)(𝑥) = ℎ(𝑥)𝑔(𝑥) = (4 − 5𝑥)(√𝑥) = 4√𝑥 − 5𝑥√𝑥 Domain: {𝑥|𝑥 ≥ 0}

e. Domain: {𝑥|𝑥 > 0 𝑎𝑛𝑑 𝑥 ≠ 2}

Example 2: Given 𝑓(𝑥) = 2𝑥, 𝑔(𝑥) = 𝑥 + 4, and ℎ(𝑥) = 5 − 𝑥3, evaluate

a. (𝑓 + 𝑔)(2) b. (ℎ − 𝑔)(2) c. (𝑓ℎ)(2) d.


Solution: Since the given value of x is 2 for all functions, we have
𝑓(2) = 2(2) = 4
𝑔(2) = 2 + 4 = 6 ℎ(2)
3
= 5 − (2) = 5 − 8 = −3
So, by definitions,
a.(𝑓 + 𝑔)(2) = 𝑓(2) + 𝑔(2) = 4 + 6 = 10
b.(ℎ − 𝑔)(2) = ℎ(2) − 𝑔(2) = −3 − 6 = −9
c. (𝑓ℎ)(2) = 𝑓(2) ∙ ℎ(2) = 4(−3) = −12

d.

¤ Composition of Functions
We can derive complex functions from simple functions such as money, location, and time
using the process of composition. In this process, the output of one function becomes
the input of another.
COMPOSITION OF FUNCTIONS
Given two functions𝑓 and 𝑔 , the composite functions𝑓 ∘ 𝑔 (also called the
composition of𝑓 and 𝑔 ) is defined by
( 𝑓 ∘ 𝑔 )( 𝑥 ) = 𝑓(𝑔 ( 𝑥 ))

The domain of 𝑓 ∘ 𝑔 is the set of all 𝑥 in the domain of 𝑔 such that 𝑔(𝑥) is in the domain
of 𝑓. In other words, (𝑓 ∘ 𝑔)(𝑥) is defined whenever both 𝑔(𝑥) and 𝑓(𝑔(𝑥)) are defined.
98
Prepared by: Rennil S. Bornia MST-Math
Self- Instructional Module on College and Advance Algebra & Advanced Mathematics

Example 2: Let 𝑓(𝑥) = 𝑥2 and 𝑔(𝑥) = 𝑥 − 3


a. Find the functions 𝑓 ∘ 𝑔 and 𝑔 ∘ 𝑓 and their
domains.
b. Find (𝑓 ∘ 𝑔)(5) and (𝑔 ∘ 𝑓)(7). Solution:
a. (𝑓 ∘ 𝑔)(𝑥) = 𝑓(𝑔(𝑥)) = 𝑓(𝑥 − 3) = (𝑥 − 3)2
(𝑔 ∘ 𝑓)(𝑥) = 𝑔(𝑓(𝑥)) = 𝑔(𝑥2) = 𝑥2 − 3 The
domains of 𝑓 ∘ 𝑔 and 𝑔 ∘ 𝑓 are ℝ.
b. (𝑓 ∘ 𝑔)(5) = 𝑓(𝑔(5)) = 𝑓(2) = 22 = 4
(𝑔 ∘ 𝑓)(7) = 𝑔(𝑓(7)) = 𝑔(49) = 49 − 3 = 46
Example 3: If , find the following functions and their
domains. a. 𝑓 ∘ 𝑔 b. 𝑔 ∘ 𝑓 c. 𝑓 ∘ 𝑓 d. 𝑔 ∘ 𝑔
Solution:
a.
The domain of 𝑓 ∘ 𝑔 is {𝑥|2 − 𝑥 ≥ 0} = {𝑥|𝑥 ≤ 2} = (−∞, 2].
b.
For to be defined, we must have to be defined, we must have
, that is, . Thus, we have 0 ≤ 𝑥 ≤ 4, so the domain of 𝑔 ∘ 𝑓 is the closed
interval [0,4].
c.
The domain of 𝑓 ∘ 𝑓 is [0, ∞).
d.
This expression is defined when both . The first inequality means
𝑥 ≤ 2, and the second is equivalent to . Thus, −2 ≤ 𝑥 ≤ 𝑥,
so the domain of 𝑔 ∘ 𝑔 is [−2,2].

Example 4: Let 𝑓(𝑥) = 3𝑥 − 2 and 𝑔(𝑥) = 𝑥2 + 2𝑥. Evaluate each of the following
expressions.
a. 𝑔(𝑓(3)) b. 𝑓(𝑔(−4)) c. (𝑔 ∘ 𝑓)(2) d. (𝑓 ∘ 𝑔)(2)
Solution:
a. Because 𝑓(3) = 3(3) − 2 = 7, we have 𝑔(𝑓(3)) = 𝑔(7) = 72 + 2 ∙ 7 = 63. So
𝑔(𝑓(3)) = 63.
b. Because 𝑔(−4) = (−4)2 + 2(−4) = 8, we have 𝑓(𝑔(−4)) = 𝑓(8) = 3(8) − 2 =
22. So, 𝑓(𝑔(−4)) = 22.
c. Because (𝑔 ∘ 𝑓)(2) = 𝑔(𝑓(2)), we first find 𝑓(2): 𝑓(2) = 3(2) − 2 = 4
Because 𝑓(2) = 4, we have (𝑔 ∘ 𝑓)(2) = 𝑔(𝑓(2)) = 𝑔(4) = 42 + 2(4) = 24.
So, (𝑔 ∘ 𝑓)(2) = 24.
d. Because (𝑓 ∘ 𝑔)(2) = 𝑓(𝑔(2)), we first find 𝑔(2): 𝑔(2) = 22 + 2(2) = 8
Because 𝑔(2) = 8, we have (𝑓 ∘ 𝑔)(2) = 𝑓(𝑔(2)) = 𝑓(8) = 3(8) − 2 = 22.
So, (𝑓 ∘ 𝑔)(2) = 22.

99
Prepared by: Rennil S. Bornia MST-Math
Self- Instructional Module on College and Advance Algebra & Advanced Mathematics

¤ Decomposition of a Function
 Decomposing a function is the reverse process of composition of functions.
Example 5: Let 𝑓(𝑥) = 3𝑥 + 6 and 𝑓(𝑔(𝑥)) = 3𝑥 − 6, find 𝑔(𝑥).
Solution:
𝑓(𝑥) = 3𝑥 + 6
𝑓(𝑔(𝑥)) = 3(𝑔(𝑥)) + 6
3𝑥 − 6 = 3(𝑔(𝑥)) + 6
3𝑥 − 6 − 6 = 3(𝑔(𝑥))
3𝑥 − 12 = 3(𝑔(𝑥))

𝑔(𝑥) = 𝑥 − 4
Example 6: Given 𝑓(𝑔(𝑥)) = 3𝑥2 + 12𝑥 + 16 and 𝑔(𝑥) = 𝑥 + 2, find 𝑓(𝑥).
Solution: Notice that the function asked for is the outer function. To solve for 𝑓(𝑥), first derive
𝑥 in 𝑔(𝑥).
Then, 𝑥 = 𝑔(𝑥) − 2.
Substitute the expression 𝑔(𝑥) − 2 for every 𝑥 in 𝑓(𝑔(𝑥)).
𝑓(𝑔(𝑥)) = 3𝑥2 + 12𝑥 + 16
= 3[𝑔(𝑥) − 2]2 + 12[𝑔(𝑥) − 2] + 16
𝑓(𝑥) = 3(𝑥 − 2)2 + 12(𝑥 − 2) + 16
𝑓(𝑥) = 3(𝑥2 − 4𝑥 + 4) + 12𝑥 − 24 + 16
𝑓(𝑥) = 3𝑥2 − 12𝑥 + 12 + 12𝑥8
𝑓(𝑥) = 3𝑥2 + 4

D. Applications
Learning Task 5.1.3: Justify Your Answer!!!
Instructions:
1. Suppose you work forty hours a week at a mall’s bookstore. Your salary every week is
₱1,500 with an additional 2. 5% commission on sales over ₱25,000. Assume that you
sell enough this week to get the commission. Given the functions 𝑓(𝑥) = 0.025𝑥 and
𝑔(𝑥) = 𝑥 − 25,000, which of these expressions (𝑓 ∘ 𝑔)(𝑥) and (𝑔 ∘ 𝑓)(𝑥) represents your
commission? Show your work that verifies your decision.
2. Using the derived function, compute for your commission if the amount of sales reaches
₱55,000; ₱100,000; and ₱250,000. Show your solutions.

3. Is there a difference between (𝑓 ∘ 𝑔)(𝑥) and (𝑔 ∘ 𝑓)(𝑥)? Justify your answer.


100
Prepared by: Rennil S. Bornia MST-Math
Self- Instructional Module on College and Advance Algebra & Advanced Mathematics

4. While (𝑓 + 𝑔)(𝑥) = 𝑓(𝑥) + 𝑔(𝑥), does it follow that 𝑓(𝑎 + 𝑏) = 𝑓(𝑎) + 𝑓(𝑏)? Justify your
answer.

IV. Assessment
Learning Task 5.1.4: Evaluation Quiz

Functions 𝑓 and 𝑔 are given. Find each of the following functions and state its domain.

a. 𝑓 + 𝑔 b. 𝑓 − 𝑔 c. 𝑓 ∙ 𝑔 d.

Evaluate the indicated function, where 𝑓(𝑥) = 𝑥2 − 3𝑥 + 2 and 𝑔(𝑥) = 2𝑥 − 4.

5. (𝑓 − 𝑔)(−3) ()

Functions 𝑓 and 𝑔 are given. Find each composite function and describe its domain.
a. 𝑓 ∘ 𝑔 b. 𝑔 ∘ 𝑓
() ()

Evaluate each composite function, where 𝑓(𝑥) = 2𝑥 + 3; 𝑔(𝑥) = 𝑥2 − 5𝑥; 𝑎𝑛𝑑 ℎ(𝑥) = 4 − 3𝑥2.
11. (𝑔 ∘ 𝑓)(4) () 13. (𝑓 ∘ 𝑔)(−3)

Find the indicated function.


14. Given 𝑓(𝑥) = 𝑥 + 6 and 𝑓(𝑔(𝑥)) = 3𝑥 + 6, find 𝑔(𝑥).
15. Given 𝑓(𝑥) = 𝑥2 + 1 and 𝑓(𝑔(𝑥)) = 𝑥4 + 4𝑥2 + 5, find 𝑔(𝑥).
16. Given .

Show that (𝑓 ∘ 𝑔)(𝑥) = (𝑔 ∘ 𝑓)(𝑥).


17. 𝑓(𝑥) = 2𝑥 + 3; 𝑔(𝑥) = 5𝑥 + 12

Show that

(𝑓 ∘ 𝑔)(𝑥) = 𝑥 and (𝑔 ∘ 𝑓)(𝑥) = 𝑥. 3

101
Prepared by: Rennil S. Bornia MST-Math
Self- Instructional Module on College and Advance Algebra & Advanced Mathematics

I. Enhancement
Learning Task 5.1.5: Let’s Create!
Generally, the composition of functions is not commutative, meaning, (𝑓 ∘ 𝑔)(𝑥) ≠ (𝑔
∘ 𝑓)(𝑥). However, there are some cases that (𝑓 ∘ 𝑔)(𝑥) = (𝑔 ∘ 𝑓)(𝑥) holds true. Examples of
this are for the functions 𝑓(𝑥) = 2𝑥 + 3 and 𝑔(𝑥) = 5𝑥 + 12. Other examples of this are for
functions and
.
Now it is your turn to find two functions whose compositions are equal. Give at least
three pairs of functions 𝑓(𝑥) and 𝑔(𝑥) such that (𝑓 ∘ 𝑔)(𝑥) = (𝑔 ∘ 𝑓)(𝑥) is true.

II. References
Ratti, J. M., & Marcus, S. (2014). College Algebra and Trigonometry (3rd Ed.). Pearson
Education Limited. Stewart, J., Redlin, L., & Watson, S. (2015). Algebra and trigonometry.
Cengage Learning.

Lesson 5.2: Rational Functions


I. Desired Learning Outcomes

At the end of the lesson, you will be able to:


• define the domain and asymptotes of rational functions;
102
Prepared by: Rennil S. Bornia MST-Math
Self- Instructional Module on College and Advance Algebra & Advanced Mathematics

• determine the domain of rational functions;


• find vertical and horizontal asymptotes;
• match the rational functions to its corresponding graphs ;  solve problems
involving rational functions; and  transform graphs of rational functions.

II. Lesson Overview

Rational function is the name given to a function which can be represented as the quotient
of polynomials, just as a rational number is a number which can be expressed as a
quotient of whole numbers. This lesson deals with rational functions and their graphs. This
also includes discussion in finding the domain and asymptotes of the rational functions.

III. Course Contents and Learning Experiences

A. Activity
Learning Task 5.2.1: Warm-Ups
True or False? Explain your answer.
1.The domain of .
2.The domain of .
3.The domain of .
4.The line 𝑥 = 2 is the only vertical asymptote for the graph of .

5.The x-axis is a horizontal asymptote for the graph of .


6.The x-axis is a horizontal asymptote for the graph of .

B. Analysis
Learning Task # 5.2.2: Think and
Discuss Answer the following questions:
1.What is a rational function?
2.What is the domain of a rational function?
3.What is a vertical asymptote?
4.What is a horizontal asymptote?

C. Abstraction

103
Prepared by: Rennil S. Bornia MST-Math
Self- Instructional Module on College and Advance Algebra & Advanced Mathematics

Examples of rational functions are .


¤ Finding the Domain of a Rational Function Example 1: Find
the domain of each rational function.

a. b. c.
Solution: We eliminate all 𝑥 for which 𝐷(𝑥) = 0.

a.𝐷(𝑥) = 𝑥 − 1 = 0, if 𝑥 = 1. The domain of in set-builder notation is {𝑥|𝑥 ∈ ℝ, 𝑥 ≠


1}, or in interval notation, (−∞, 1) ∪ (1, ∞).
b.The domain of 𝑔 is the set of all real numbers 𝑥 for which the denominator 𝐷(𝑥) is nonzero.
𝐷(𝑥) = 𝑥2 − 6𝑥 + 8 = 0
(𝑥 − 2)(𝑥 − 4) = 0
𝑥−2=0 or 𝑥−4=0
𝑥=2 𝑥=4
The domain of in set-builder notation is {𝑥|𝑥 ∈ ℝ, 𝑥 ≠ 2, 4}, or in interval
notation, (−∞, 2) ∪ (2, 4) ∪ (4, ∞).

c. The domain of in set-builder notation is {𝑥|𝑥 ∈ ℝ, 𝑥 ≠ 2}, or in interval notation


(−∞, 2) ∪
(2, ∞).

¤ Vertical and Horizontal Asymptotes

To better understand the next illustrations, here are the used notations and their definitions.
Notation Definition
𝑥 → 𝑎− 𝑥 approaches 𝑎 from the left (through values less than 𝑎)
𝑥 → 𝑎+ 𝑥 approaches 𝑎 from the right (through values greater than
𝑎)
𝑓(𝑥) → ∞ 𝑓 (𝑥) increases without bound (can be made as large
104
Prepared by: Rennil S. Bornia MST-Math
Self- Instructional Module on College and Advance Algebra & Advanced Mathematics

positive desired)
decreases without bound (can be made as large negative
desired)
Shown below are some illustrations as vertical asymptotes of the graph of 𝑓 represented by
dashed lines.

Example 2. Find the vertical asymptotes of each rational function.

a. b.
Solution:
a.To find the vertical asymptotes, determine the real zeros of
the denominator. The denominator 𝑥2 + 1 has no real zeros,
so the graph of 𝑓 has no vertical asymptotes. The graph is
shown on the right.

b.he denominator 𝑥2 − 𝑥 − 6 = (𝑥 − 3)(𝑥 + 2) has zeros of 3


and -2.
The numerator has no common factors with the
denominator, so 𝑥 = 3 and 𝑥 = −2 are both vertical
asymptotes of the graph of 𝑔 as shown on the right.

105
Prepared by: Rennil S. Bornia MST-Math
Self- Instructional Module on College and Advance Algebra & Advanced Mathematics

Example 3. Find the horizontal asymptote of each rational function.


2𝑥 +3 4𝑥 2 +1 𝑥 3 +1
𝑓 (𝑥 ) = b. 𝑔 ( 𝑥 ) = c. ℎ ( 𝑥 ) =
𝑥 2 +1 3𝑥 2 𝑥−2

a. The degree of the numerator2𝑥 + 3 is less than the degree of the


2+ 1
. By the Theorem on Horizontal Asymptotes, the -xaxis
is the horizontal asymptote of𝑓. The graph is shown on the right.

2
+ 1 and the denominator3𝑥 2 of 𝑔 are both of
4
degree 2. By the Theorem on Horizontal Asymptotes, the line𝑦 = is 3
the horizontal asymptote of𝑔. See the graph of 𝑔 on the right.

c. The degree of the numerator𝑥 3 + 1 is larger than the degree of the

a.
Solution:

denominator 𝑥

b.The numerator 4𝑥

106
Prepared by: Rennil S. Bornia MST-Math
Self- Instructional Module on College and Advance Algebra & Advanced Mathematics

denominator 𝑥 − 2, so by the Theorem on Horizontal Asymptotes, the graph of ℎ has no


horizontal asymptotes.

¤ Graph of Rational Functions

D. Applications
Learning Task 5.2.3:
Instructions: Solve each problem
1.The number of deer P at any time t (in years) in a federal game reserve is given by

a.Find the number of deer when t is 15, 70, 100.


b.Find the horizontal asymptote of the graph of 𝑦 = 𝑃(𝑡).
c. Sketch a graph of the function.
d.According to the model, what is the largest possible deer population?

107
Prepared by: Rennil S. Bornia MST-Math
Self- Instructional Module on College and Advance Algebra & Advanced Mathematics

e.Write a paragraph that explains the parts of the graph in terms of the situation. What do
you know about the population? Are all parts of the graph valid for the model? Why or
why not?
2.Sunsport Recreation, Inc. wants to build a rectangular swimming pool with a pool surface
of 1200 square feet. They are required to have a walk of uniform width 2.5-ft surrounding
the pool. Let x be the length of one side of the swimming pool.
a.Draw a picture of the pool.
b.Express the area covered by the pool and sidewalk as a function of x.
c. Sketch a graph of the function,
d.Find the dimensions that cover the least area. (use your calculator if necessary)

3.Sarah rode her bike 10 miles from her home in Springfield, Illinois, and then took a 35-
mile trip by car to complete the trip from Springfield to Decatur. Assume the average rate
of the car was 40 mph faster than the average rate of the bike.
a.Express the total time t required to complete the 45-mile trip (bike and car) as a function
of the rate x of the car.
b.Use a graphical method to find the rate of the car if the total time of the trip was 1 hour.
Confirm your answer algebraically.

IV. Assessment
Learning Task 5.2.4: Evaluation Quiz
Determine the domain of the rational functions.

2.

4.

Find all vertical asymptotes of each rational function.

Find the horizontal asymptote of each rational function.

Match the rational function with its graph.

108
Prepared by: Rennil S. Bornia MST-Math
Self- Instructional Module on College and Advance Algebra & Advanced Mathematics

2 𝑥−2 1
16. 𝑓 ( 𝑥 ) = 17. 𝑓 ( 𝑥 ) = 18. 𝑓 ( 𝑥 ) =
𝑥−3 𝑥+3 𝑥 2 −2𝑥
𝑥 𝑥2
19. 𝑓 ( 𝑥 ) = 20. 𝑓 ( 𝑥 ) =
𝑥 2 +1 𝑥 2 −4

V. Enhancement
Learning Task 5.2.5: Let’s Create!
Instructions:
Find a pair and visit this link: https://www.youtube.com/watch?v=oDVUStRWXpM
Watch YouTube video about “Transformations with Rational Functions”, then do the
following:

A.Use transformations of the graph of to graph the following rational function.

Note: If the rational function is improper like , use long division method to make
it proper 𝑓(𝑥) =
.
For example, can be written as
because

109
Prepared by: Rennil S. Bornia MST-Math
Self- Instructional Module on College and Advance Algebra & Advanced Mathematics

B.Find an equation of a rational function having the given asymptotes, intercepts, and
graph.

VI. References
Ratti, J. M., & Marcus, S. (2014). College Algebra and Trigonometry (3rd Ed.). Pearson
Education Limited.Stewart, J., Redlin, L., & Watson, S. (2015). Algebra and trigonometry.
Cengage Learning.
https://www.youtube.com/watch?v=oDVUStRWXpM

Lesson 5.3: Inverse Functions


I. Desired Learning Outcomes

At the end of the lesson, you will be able to:


• determine if the function is one-to-one by using horizontal test or by solving
algebraically;
• verify if two functions are inverses of each other;
• find the inverse function;
• find the domain and range of inverse function;  solve problems involving
inverse functions; and  the graph of an inverse function.

II. Lesson Overview


The goals of this lesson are to present inverses in a slightly different context: to help you
understand that when a function and its inverse are composed, they undo each other, and
to review a process for creating the inverse of a function algebraically.

III. Course Contents and Learning Experiences

A.Activity
Learning Task 5.3.1: The graph of a function is given. Use the horizontal-line test to
determine whether the function is one-to-one.

110
Prepared by: Rennil S. Bornia MST-Math
Self- Instructional Module on College and Advance Algebra & Advanced Mathematics

2. 3. 4.

6. 7. 8.

1.

5.

B.Analysis
Learning Task # 5.3.2: Think and
Discuss 1. What is the inverse of a
function?
2. What is the domain of 𝑓−1?
3. What is the range of 𝑓−1?
4. What does the −1 in 𝑓−1 mean?
5. What is a one-to-one function?
6. What is the horizontal-line test?

C. Abstraction

¤ One-to-One Functions
The inverse of a function is a rule that acts on the output of the function and produces the
corresponding input. So the inverse “undoes” or reverses what the function has done.
Not all functions have inverses; those that do are called one-to-one.

111
Prepared by: Rennil S. Bornia MST-Math
Self- Instructional Module on College and Advance Algebra & Advanced Mathematics

DEFINITION OF ONE-TO-ONE FUNCTIONS


The function, 𝑓(𝑥), is a one to one function when each element of its range
will be returned by one unique element from its domain. This means that for
every value of 𝑥, there will be a unique value of 𝑦 or 𝑓(𝑥).

HORIZONTAL TEST
A function is one-to-one if and only if no horizontal line intersects its graph
more than once.

DEFINITION OF ONE-TO-ONE FUNCTIONS


The function, 𝑓(𝑥), is a one to one function when each element of its range
will be returned by one unique element from its domain. This means that for
every value of 𝑥, there will be a unique value of 𝑦 or 𝑓(𝑥).

Example 1: Determine which of the following functions are one-to-one.


a. 𝑓(𝑥) = 2𝑥 + 5 b. 𝑔(𝑥) = 𝑥2 − 1 c.
Solution:

112
Prepared by: Rennil S. Bornia MST-Math
Self- Instructional Module on College and Advance Algebra & Advanced Mathematics

¤ The Inverse of a Function


One-to-one functions are important because they are precisely the functions that possess
inverse functions according to the following definition.
DEFINITION OF THE INVERSE OF A FUNCTION
Let 𝑓 be a one-to-one function with domain 𝐴 and range 𝐵. Then its
inverse function 𝑓−1 has domain 𝐵 and range 𝐴 and is defined by
𝑓−1(𝑦) = 𝑥 𝑓(𝑥) = 𝑦 for any 𝑦 in 𝐵.

FINDING THE INVERSE OF A FUNCTION


If a function is a one-to-one, then its inverse can be obtained by the
following simple steps:
1.Replace 𝑓(𝑥) by 𝑦.
2.Interchange 𝑥 and 𝑦 variables.
3.Solve for 𝑦 in terms of 𝑥.
4.Replace 𝑦 by 𝑓−1(𝑥).
Example. Find the inverse of the following one-to-one functions:
(a) 𝑓(𝑥) = 𝑥 − 4
Solution:
(a) 𝑓(𝑥) = 𝑥 − 4 (b)
Step 1: 𝑦=𝑥−4
Step 2: 𝑥=𝑦−4 Step 1:
Step 3: 𝑦=𝑥+4 Step 2:
−1
Step 4: 𝑓 (𝑥) = 𝑥 + 4
Step 3:
𝑦 = −3𝑥 + 3
−1
Step 4: 𝑓 (𝑥) = −3𝑥 + 3
(c)
Step 1:
Step 2:
Step 3: 𝑥𝑦 = 2𝑦 + 1
𝑥𝑦 − 2𝑦 = 1
𝑦(𝑥 − 2) = 1

Step 4:

INVERSE FUNCTION PROPERTY


Let 𝑓 denote a one-to-one function. Then
1. 𝑓−1(𝑓(𝑥)) = 𝑥 for every 𝑥 in the domain of 𝑓.
113
Prepared by: Rennil S. Bornia MST-Math
Self- Instructional Module on College and Advance Algebra & Advanced Mathematics

2. 𝑓(𝑓−1(𝑥)) = 𝑥 for every 𝑥 in the domain of 𝑓−1.


Further, if 𝑔 is any function such that (for all values of 𝑥 in the domain of
the inner function)
𝑓(𝑔(𝑥)) = 𝑥 and 𝑔(𝑓(𝑥)) = 𝑥, then 𝑔 = 𝑓−1.

Example. Verifying That Two Functions Are Inverses


Show that are inverses of each other.
Solution:
(𝑓 ∘ 𝑔)(𝑥) = 𝑓(𝑔(𝑥)) (𝑔 ∘ 𝑓)(𝑥) = 𝑔(𝑓(𝑥))
= 𝑔(2𝑥 + 3)
𝑥−3
=𝑥
=𝑥
Because 𝑓(𝑔(𝑥)) = 𝑔(𝑓(𝑥)) = 𝑥, 𝑓 and 𝑔 are inverses of each other.

¤ Domain and Range of Inverse Functions


An interesting relationship exists between the domain and range of a function 𝑓 and those
of the inverse functions. They are directly related functions due to the fact that there are
corresponding points in the graphs of 𝑓 and 𝑓−1 that have their coordinates reversed. If
(𝑥, 𝑦) is in the graph of 𝑓, then (𝑦, 𝑥) is in the graph of 𝑓−1. This implies that the domain
values of 𝑓−1 are the range values of 𝑓. Similarly, the range values of 𝑓−1 are the domain
values of 𝑓.

To summarize, we have the following:


1.Domain of 𝑓 = Range of 𝑓−1
2.Range of 𝑓 = Domain of 𝑓−1

D. Applications
Learning Task 5.3.3: Think-Pair-Solve
Instructions:
1. The function is used to convert 𝑥 degrees Fahrenheit to an equivalent
Celsius temperature. Find 𝑓 and explain what does it represent.
−1

2. A clothing merchant uses the function to determine the retail selling


price 𝑆, in dollars, of a winter coat for which she has paid a wholesale price of 𝑥 dollars.
a. The merchant paid a wholesale price of $96 for a winter coat. Use 𝑆 to
determine the retail selling price she will charge for this coat.
b. Find 𝑆−1 and use it to determine the merchant’s wholesale price for a coat that
retails at $399.
114
Prepared by: Rennil S. Bornia MST-Math
Self- Instructional Module on College and Advance Algebra & Advanced Mathematics

3. The function 𝑠(𝑥) = 2𝑥 + 24 can be used to convert a U.S. women’s shoe size into an Italian
women’s shoe size. Determine the function 𝑠−1(𝑥) that can be used to convert an Italian
women’s shoe size to its equivalent U.S. shoe size.
4. Anwar is a short-order cook in a diner. He is paid ₱200 per hour plus 5% of all food sales per
hour. His average hourly wage 𝑤 in terms of the food sales of 𝑥 pesos is 𝑤 = 200 + 0.05𝑥.
a. Write the inverse function. What does it mean?
b. Use the inverse function to estimate the hourly sales at the diner if Anwar averages
₱600 per hour.

IV. Assessment
Learning Task 5.3.4: Evaluation Quiz
Determine whether the function is one-to-one.
1. 𝑓(𝑥) = −2𝑥 + 4 2. 𝑓(𝑥) = 3𝑥 − 2 3. 𝑔(𝑥) = 𝑥2 − 2𝑥
4. ℎ(𝑥) = 𝑥4 + 5

Show that 𝑓 and 𝑔 are inverses of each other by verifying that 𝑓(𝑔(𝑥)) = 𝑥 = 𝑔(𝑓(𝑥))

10. 𝑓(𝑥) = 𝑥3 + 1; 𝑔(𝑥) = (𝑥 − 1)1⁄3

Find the inverse function


of 𝑓.
11. 𝑓(𝑥) = 4𝑥 + 7

V. Enhancement
Learning Task 5.3.5: Let’s Create!
Watch YouTube video with this link: https://www.youtube.com/watch?
v=veMUDREk44c&t=650s about Graphs of Inverse Functions. Then do the following:
The graph of a function 𝑓 is given. Sketch the graph of 𝑓−1 and find the domain and range
of 𝑓 and 𝑓−1.

115
Prepared by: Rennil S. Bornia MST-Math
Self- Instructional Module on College and Advance Algebra & Advanced Mathematics

VI. References
Ratti, J. M., & Marcus, S. (2014). College Algebra and Trigonometry (3rd Ed.). Pearson
Education Limited.Stewart, J., Redlin, L., & Watson, S. (2015). Algebra and trigonometry.
Cengage Learning.
https://www.youtube.com/watch?v=veMUDREk44c&t=650s

116
Prepared by: Rennil S. Bornia MST-Math
Self- Instructional Module on College and Advance Algebra & Advanced Mathematics

UNIT 6: EXPONENTIAL AND LOGARITHMIC


FUNCTIONS
Unit Overview

Exponential and logarithmic functions


have varieties of applications in several
fields like biology, natural sciences,
business, and ecology. An exponential
function is used to model the
population growth of organism, growth
of money in a bank, as well as decay
of elements. Logarithmic functions can
be used to scale very large (or very
small) numbers so that they are easier
to comprehend.

This unit introduces other functions,


namely, exponential and logarithmic
functions. In Lesson 6.1, you will identify and evaluate exponential functions, find the
domain, range and properties of the graph of this kind of function. In Lesson 6.2 you will
learn to write exponential form to logarithmic form and vice versa. You will also evaluate
the function and find its domain and identify the properties of its graph.

Moreover, at the end of this unit, you will be able to:


 determine the nature of exponential and logarithmic functions
 evaluate exponential and logarithmic functions
 find the domain and range of exponential and logarithmic functions
 match the exponential or logarithmic functions to its corresponding graph
 convert exponential expressions into logarithmic expressions and vice
versa
 solve problems involving exponential and logarithmic functions
 transform graphs of exponential and logarithmic functions

117
Prepared by: Rennil S. Bornia MST-Math
Self- Instructional Module on College and Advance Algebra & Advanced Mathematics

Lesson 6.1: Exponential Functions and Their Applications


I. Desired Learning Outcomes

At the end of the lesson, you will be able to:


• determine the nature of exponential functions
• evaluate exponential functions
• find the domain and range of exponential functions
• match the exponential functions to its corresponding graph
• find the zeros of exponential functions
• solve problems involving exponential functions
• transform graphs of exponential functions

II. Lesson Overview

In the previous lessons, you learned variety of functions. In this lesson, you will be
introduced with exponential functions by investigating paper folding activity which leads you
to compare exponential growth and decay. Moreover, you will also learn the applications of
exponential functions to real life and graph exponential function by transformations.

III. Course Contents and Learning Experiences

A. Activity
Learning Task 6.1.1: Let’s Investigate
1.Get a piece of paper and fold it in half. The fold divides the paper into two regions,
each of which has half the area of the paper.
2.Fold the paper in again according to the number of folds shown in the table below.
3.Then, complete the table below showing the number of folds, the number of regions into
which the paper has been folded, and the fraction of the paper’s area that each region
has.

Number of folds 0 1 2 3 4 5
Number of regions 1 2
Fractional area of each 1 ½
region

B. Analysis
Learning Task 6.1.2:
Based on the paper folding activity above, answer the following:

118
Prepared by: Rennil S. Bornia MST-Math
Self- Instructional Module on College and Advance Algebra & Advanced Mathematics

4.Make two graphs of the data in the table. The first graph will have ordered pairs of the
form (fold number, number of regions) and the second will have ordered pairs of the
form (fold number, fractional area of each region).

5.The first graph is an example of exponential growth. Write an equation for the graph.
6.Use the equation in number 5 to determine the number of regions there would be after 8
folds.
7.The second graph is an example of exponential decay. Write an equation for the graph.
8.Use the equation in number 7 to determine the fractional area of each region after 8
folds.
9.Multiply the exponential expressions from numbers 5 and 7. Explain why the product
should be 1.
10. What relationships exist between exponential growth and exponential decay when a
piece of paper is folded repeatedly?

¤ Exponential Functions

¤ Identifying and Evaluating Functions


How do we then distinguish phenomenon which is exponential in nature from others
which are not? While there are many ways to do so, one convenient way is to check the
pattern that may exist among the values of the dependent variable y when the
independent variable x are integers arranged consecutively. if there is a common ratio
between two consecutive values, then we can say that the function is exponential.
Here are some examples showing the existence of common ratio in a function which is
exponential in nature. Before checking the pattern on the values of the dependent
variable y, check first if the values of the independent variable x are integers arranged
consecutively.

Example 1. Identifying Functions


Does each table represent an exponential function?

119
Prepared by: Rennil S. Bornia MST-Math
Self- Instructional Module on College and Advance Algebra & Advanced Mathematics

Solution:

Example 2. Evaluating Exponential Functions


Evaluate 𝑓(𝑥) = 3𝑥 at 𝑥 = 2, 𝑥 = −4, and 𝑥 = 𝜋.
Solution:
𝑓(2) = 32 = 9

𝑓(𝜋) = 3𝜋 = 33.1415927 ≈ 31.54428 Evaluate with the aid of a calculator

Example Find the following:


a. b. 𝑓(−3) c. 𝑔(3) d. ℎ(2)
Solution:
3
a. 𝑓 ( 2) = 2 3⁄2 = √2 3 = √8 = 2√2
1 1
b. 𝑓 ( −3 ) = 2 −3 = 23
=
8
1 1−3 1 −2
c. 𝑔 ( 3 ) = ( 4) = ( ) = 4 2 = 16
4
d. ℎ(2) = −32 = −9 Note that −32 ≠ (−3)2.

¤ Graphs of Exponential Functions


The graph of a function 𝑦 = 𝑎𝑏𝑥 is a vertical stretch or shrink by a factor of
|𝑎| of the graph of the parent function 𝑦 = 𝑏𝑥. When 𝑎 < 0, the graph is also
reflected in the x-axis. The y-intercept of the graph of 𝑦 = 𝑎𝑏𝑥 is 𝑎.

Example 3. Graphing 𝑦 = 𝑎𝑏𝑥 when 𝑏 > 1


Graph 𝑓(𝑥) = 4(2)𝑥. Compare the graph to the graph of the parent function.
Describe the domain and range of 𝑓.
Solution:

120
Prepared by: Rennil S. Bornia MST-Math
Self- Instructional Module on College and Advance Algebra & Advanced Mathematics

Example 4: Graphing when 0 < 𝑏 < 1


Graph . Compare the graph to the graph of the parent function. Describe
the domain and range of 𝑓.
Solution:

Example 5: Graphing 𝑦 = 𝑎𝑏𝑥−ℎ + 𝑘


Graph 𝑓(𝑥) = 4(2)𝑥−3 + 2. . Describe the domain and range.
Solution:

¤ Natural Exponential Function

121
Prepared by: Rennil S. Bornia MST-Math
Self- Instructional Module on College and Advance Algebra & Advanced Mathematics

The irrational number 𝜋 is often used in applications that involve circles. Another irrational
number, denoted by the letter e, is useful in many applications that involve growth or
decay.

The letter 𝑒 was chosen in honor of the Swiss mathematician Leonhard Euler. He was
able to compute the

𝑛
value of 𝑒 to several decimal places by evaluating for large values of 𝑛, as shown in table
below.

The value of e accurate to eight decimal places is 2.71828183.


The base of an exponential function can be any positive real number other than 1. The
number 10 is a convenient base to use for some situations, but we will see that the
number 𝑒 is often the best base to use in real-life applications. The exponential function
with 𝑒 as the base is known as the natural exponential function.

¤ Exponential Equations

In Unit 5, we used the horizontal-line test to determine whether a function is one-to-one.


Because no horizontal line can cross the graph of an exponential function more than
once, exponential functions are one-to-one functions. For an exponential function one-
to-one means that if two exponential expressions with the same base are equal, then
the exponents are equal. If 2𝑥 = 2𝑦 then 𝑥 = 𝑦.
122
Prepared by: Rennil S. Bornia MST-Math
Self- Instructional Module on College and Advance Algebra & Advanced Mathematics

Example 6: Solve each equation.

a. 22𝑥−1 = 8 b. c.
Solution:
a.Because 8 is 23, we can write each side as a power of the same base, 2:

b. Because , we can write


4 = 2 2 , 8 = 2 3 , and 256 = 2 8 each side as a power of the
same base, 2.

Write each side as a power of the same base


−2 2𝑦+1 3𝑦 8
(2 ) (2 ) = 2 Negative exponent rule
−4𝑦−2 3𝑦 8
(2 )(2 ) = 2 Power-to-power rule
−𝑦−2 8
2 =2 Product rule
−𝑦 − 2 = 8 One-to-one property
−𝑦 = 10
𝑦 = −10
Check 𝑦 = −10 in the original equation. The solution set is {−10}.

c. Because , we can write each side as a power of the same base, 2:

123
Prepared by: Rennil S. Bornia MST-Math
Self- Instructional Module on College and Advance Algebra & Advanced Mathematics

Example 7: Find the zeros of


𝑓(𝑥) = 4𝑥 − 64.
Solution:
𝑓(𝑥) = 4𝑥 − 64
0 = 4𝑥 − 64 Let 𝑓(𝑥) = 0
𝑥
4 = 64 Addition Property of Equality
𝑥 3
4 =4 Express 64 as a
power of 4 𝑥=3 One-
to-one property Therefore, the zero of
the function is 𝑥 = 3.

¤ Applications

The total amount accumulated after t years, denoted by A, is also called the future value of
the investment.

Example 6: Using Different Compounding Periods to Compare Future Values


If $100 is deposited in a bank that pays 5% annual interest, find the
future value A after one year if the interest is compounded (i) Annually.
(ii) Semiannually.
(iii)Quarterly.
(iv) Monthly.
124
Prepared by: Rennil S. Bornia MST-Math
Self- Instructional Module on College and Advance Algebra & Advanced Mathematics

(v) Daily.
Solution:
In the following computations, 𝑃 = $100, 𝑟 = 0.05, and 𝑡 = 1. Only 𝑛, the number of times
interest is compounded each year, changes. Because 𝑡 = 1, 𝑛𝑡 = 𝑛(1) = 𝑛.
𝑛𝑡
(i) Annual Compounding:

(ii) Semiannual Compounding

(iii)Quarterly Compounding

(iv) Monthly Compounding

(v) Daily Compounding

Example 7: If $350 is deposited in an account paying 12% compounded continuously, then


how much is in the account at the end of 6 years and 6 months?
Solution:
Use 𝑟 = 12%, 𝑡 = 6.5 years and 𝑃 = $350 in the formula for
compounding interest continuously.
𝐴 = 𝑃𝑒𝑟𝑡 = 350𝑒(0.12)(6.5) = 350𝑒0.78 ≈ $763.52

D. Applications
Learning Task 6.1.3: Think-Pair-Solve

1.Find (a) the future value of the given principal P and (b) the interest earned in the given
period.
a. P = $3500 at 6.5% compounded annually for 13 years
125
Prepared by: Rennil S. Bornia MST-Math
Self- Instructional Module on College and Advance Algebra & Advanced Mathematics

b. P = $6240 at 7.5% compounded monthly for 12 years


c. P = $7500 at 5% compounded continuously for 10 years
d. P = $8000 at 6.5% compounded daily for 15 years

2.Find the principal P that will generate the given future value A.
a. A = $10,000 at 8% compounded annually for 10 years
b. A = $10,000 at 8% compounded quarterly for 10 years
c. A = $10,000 at 8% compounded daily for 10 years
d. A = $10,000 at 8% compounded continuously for 10 years

3.In 1626, Peter Du Minuit purchased Manhattan Island from the Native Americans for 60
Dutch guilders (about $24). Suppose the $24 was invested in 1626 at a 6% rate. How
much money would that investment be worth in
2006 if the interest was
a. simple interest.
b. compounded annually.
c. compounded monthly.
d. compounded continuously.

IV. Assessment
Learning Task 6.1.4: Evaluation Quiz

Determine whether the equation represents an exponential function or not.


1.𝑦 = 4(7)𝑥 4. 𝑦 = −3𝑥

2.𝑦 = 𝑥2
3.𝑦 = 5−𝑥 6. 𝑦 = −2𝑥

Determine whether the table represents an exponential function or not.


7. 8.

126
Prepared by: Rennil S. Bornia MST-Math
Self- Instructional Module on College and Advance Algebra & Advanced Mathematics

9. 10
.

1112
. .

1314
. .

Let . Find the following:


15. 𝑓(2) 17. 𝑔(−2) 18. 𝑔(0) 19. ℎ(−4) 20. ℎ(3)

Find the zeros of the following exponential functions:

22. ℎ(𝑥) = 8𝑥+1 − 16 23. 𝑣(𝑥) = 2𝑥+1 − 4

Examine the four functions and the graphs labeled a, b, c, and d. For each graph,
determine which function has been graphed.

26. 𝑓(𝑥) = 5𝑥 𝑔(𝑥) = 1 + 5−𝑥 𝑔(𝑥) = 5𝑥+3 ℎ(𝑥) = 5𝑥 + 3

127
Prepared by: Rennil S. Bornia MST-Math
Self- Instructional Module on College and Advance Algebra & Advanced Mathematics

V. Enhancement
Learning Task 6.1.5:
1. Watch the YouTube video on Transformations of Exponential Functions with this link:
https://www.youtube.com/watch?v=axSPECnqWWA
2. Find the domain and range of 𝑓(𝑥) and 𝑔(𝑥)
3. Graph the functions 𝑓(𝑥) and 𝑔(𝑥) in the same Cartesian plane with the aid of graphing
calculator, then describe the movement.
Describe the
Functions Domain Range Graph
movement
𝑥
a. 𝑓(𝑥) = 2 and
𝑔(𝑥) = 2𝑥 + 3
b. 𝑓(𝑥) = 3𝑥 and
𝑔(𝑥) = 3𝑥−1
c. 𝑓(𝑥) = −𝑒𝑥 and
𝑔(𝑥) = −𝑒𝑥 + 4
d. 𝑓(𝑥) = 3𝑥 and
𝑔(𝑥) = −3𝑥+2 + 3
e. 𝑓(𝑥) = 𝑒𝑥 and
𝑔(𝑥) = 𝑒3𝑥
f. 𝑓(𝑥) = 2𝑥 and
𝑔(𝑥) = −3(2𝑥−1) + 3
𝑔. 𝑓(𝑥) = 𝑒𝑥 and

4. Based on the graphs and descriptions that you made, make a general procedures or
rules in graphing transformations of exponential functions.

VI. References
Ratti, J. M., & Marcus, S. (2014). College Algebra and Trigonometry (3rd Ed.). Pearson
Education Limited.
Stewart, J., Redlin, L., & Watson, S. (2015). Algebra and trigonometry. Cengage Learning.
https://www.youtube.com/watch?v=axSPECnqWWA
Lesson 6.2: Logarithmic Functions and Their Applications
I. Desired Learning Outcomes
At the end of the lesson, you will be able to:
128
Prepared by: Rennil S. Bornia MST-Math
Self- Instructional Module on College and Advance Algebra & Advanced Mathematics

• evaluate logarithmic functions;


• find the domain and range of logarithmic functions;
• convert exponential expressions into logarithmic expressions and vice
versa;
• match the logarithmic functions to its corresponding graph;  solve
problems involving logarithmic functions; and  transform graphs of
logarithmic functions.

II. Lesson Overview


Logarithms are another way of thinking about exponents. It is important because of their
relationship to exponential functions as it can solve exponential equations and explore
properties of exponential functions. In this less, you will introduce with an activity in which
you will apply logarithm to solve the problem which is related to application of exponential
functions. You will also learn to convert exponential expression into logarithmic
expressions and vice versa. Moreover, you will have opportunity to use transformations in
graphing logarithmic functions.

III. Course Contents and Learning Experiences

A. Activity
Learning Task 6.2.1: Let’s Investigate
Read the situation below and answer the questions that follow.
Situation: Mike invested his ₱200,000 in 2016 in a bank that offers a rate of 3%
compounded annually.
Questions:
1. What function model can you derive that determines the amount Mike will have in his
investment after 𝑡 years?
2. How much interest will he receive after five years?
3. How much will he have in his investment in 2020?
4. In what year will his money double?

B. Analysis
Learning Task 6.2.2:
True or false? Explain your answer.
1. The equation 𝑎3 = 2 is equivalent to log𝑎(2) = 3.
2. If (𝑎, 𝑏) satisfies satisfies 𝑦 = log8 𝑥.
3. If 𝑓(𝑥) = 𝑎 for 𝑎 > 0 and 𝑎 ≠ 1, then 𝑓−1(𝑥) = log𝑎(𝑥).
𝑥

4. If 𝑓(𝑥) = ln(𝑥), then 𝑓1(𝑥) = 𝑒𝑥.


5. The domain of 𝑓(𝑥) = log6(𝑥) is (−∞, ∞).
6. log25(5) = 2
7. log(−10) = 1
8. log(0) = 0

129
Prepared by: Rennil S. Bornia MST-Math
Self- Instructional Module on College and Advance Algebra & Advanced Mathematics

9. 5log5(125) = 125
10. log1⁄2(32) = −5
¤ Logarithmic Function

The definition of the logarithmic function says that the two equations
(logarithmic form)
and (exponential
form)
are equivalent. For instance, because 3 must be raised to the fourth power to
obtain 81.

Example 1: Converting from Exponential to


Logarithmic Form Write each exponential equation in
logarithmic form.

a. b. c.
Solution:
a. is equivalent to .

b. is equivalent to .
c. is equivalent to .
Example 2: Converting from Logarithmic Form to
Exponential Form Write each logarithmic equation in
exponential form.
a. b. c.
Solution:
a. is equivalent to .
b. is equivalent to .
c. is equivalent to .

¤ Evaluating Logarithms
The technique of converting from logarithmic form to exponential form can be used to
evaluate some logarithms by inspection.
Example 3: Find the value of each of the following
logarithms.
a. b. c.
d. e. f.
Solution:
Logarithmic Form Exponential Form Value
130
Prepared by: Rennil S. Bornia MST-Math
Self- Instructional Module on College and Advance Algebra & Advanced Mathematics

a.
b.
c.
d.
e.

f.
¤ Basic Properties of Logarithms

Example 4: Using Basic Properties of Logarithms


Evaluate. a. log3 3 b. 5log5 7
Solution:
a.Because log𝑎 𝑎 = 1 (property 1), we have log3 3 = 1.
b.Because 𝑎log𝑎 𝑥 = 𝑥 (property 4), we have 5log5 7 = 7.

¤ Domains of Logarithmic Functions


Because the exponential function 𝑓(𝑥) = 𝑎𝑥 has domain (−∞, ∞) and range (0, ∞), its
inverse function 𝑦 = log𝑎 𝑥 has domain (0, ∞) and range (−∞, ∞). Therefore, the logarithms of
0 and of negative numbers are not defined; so expressions such as log𝑎(−2) and log𝑎(0) are
meaningless. Example 5: Finding the Domain Find the domain.
a. 𝑓(𝑥) = log3(2 − 𝑥) b.
Solution:
a.Because the domain of a logarithmic function is(0, ∞), the expression (2 − 𝑥)
must be positive. The domain of 𝑓 is the set of all real numbers 𝑥, where
2−𝑥>0
2>𝑥
So, the domain of 𝑓 is (−∞, 2).
b.The domain of 𝑔 consists of all real numbers 𝑥 for which . We can solve
this inequality using the test-point method.

From the figure above, we see that . So the domain of 𝑔 is (−∞,


−1) ∪
(3, ∞).

¤ Graphs of Logarithmic Functions


131
Prepared by: Rennil S. Bornia MST-Math
Self- Instructional Module on College and Advance Algebra & Advanced Mathematics

Example: Using Transformations on 𝑓(𝑥) = log3 𝑥


Start with the graph of 𝑓(𝑥) = log3 𝑥 and use transformations to sketch the
graph of each function.
a. 𝑓(𝑥) = log3 𝑥 + 2 b. 𝑓(𝑥) = log3(𝑥 − 1)
c. 𝑓(𝑥) = − log3 𝑥 d. 𝑓(𝑥) = log3(−𝑥)
State the domain and range and the vertical asymptote for the graph of each function.

Solution: We start with the graph of 𝑓(𝑥) = log3 𝑥 and use the transformations

¤ Common Logarithm
132
Prepared by: Rennil S. Bornia MST-Math
Self- Instructional Module on College and Advance Algebra & Advanced Mathematics

The logarithm with base 10 is called the common logarithm and is denoted by omitting the
base, so
log𝑥 = log10 𝑥

Applying the basic properties of logarithms to common logarithms, we have the following:
1.log10 = 1
2.log1 = 0
3.log10𝑥 = 𝑥, 𝑥 any real number
4.10log𝑥 = 𝑥, 𝑥 > 0

We use property (3) to evaluate the common logarithms of numbers that are powers of
10. For example,
log1000 = log103 = 3
and log0.01 = log10−2 = −2
We use a calculator to find the common logarithms of numbers that are not powers of 10
by pressing the LOG key.

¤ Natural Logarithm
In most applications in calculus and the sciences, the convenient base for logarithms is
the number 𝑒. The logarithm with base 𝑒 is called the natural logarithm and is
denoted by 𝑙𝑛 𝑥 (read “𝑒𝑙𝑙 𝑒𝑛 𝑥”) so that ln 𝑥 = log𝑒 𝑥.

Applying the basic properties of logarithms to natural logarithms, we have the following:

any real number


4. 𝑒 = 𝑥, 𝑥 > 0
𝑥

We can use property (3) to evaluate the natural logarithms of powers of e.

Example 6: Evaluating the Natural Logarithm


Function Evaluate each expression.
a. ln𝑒4 b. c. ln 3
Solution:
a.ln𝑒4 = 4
b.
c. ln 3 ≈ 1.0986123

133
Prepared by: Rennil S. Bornia MST-Math
Self- Instructional Module on College and Advance Algebra & Advanced Mathematics

¤ Applications
We recall the continuous compound interest formula:

= 𝑒 𝑟𝑡 Divide both sides by 𝑃 .


𝑃
𝐴
ln = 𝑟𝑡 Logarithmic form
𝑃

𝐴 = 𝑃𝑒𝑟𝑡 We express equation


in logarithmic form:

The exponential form is used when we need to find 𝐴 or 𝑃; logarithmic form is useful in
calculating 𝑟 or 𝑡.

Example 7: Doubling Your Money


134
Prepared by: Rennil S. Bornia MST-Math
Self- Instructional Module on College and Advance Algebra & Advanced Mathematics

a.How long will it take to double your money if it earns 6.5% compounded continuously?
b.At what rate of return, compounded continuously, would your money double in 5
years? Solution:
If 𝑃 dollars is invested and you want to double it, then the final amount 𝐴 = 2𝑃.
a. Continuous compounding formula, logarithmic form

Divide both sides by 0.065.


𝑡 = 10.66 Use a calculator.
It will take approximately 11 years to double your money.
b. Continuous compounding formula, logarithmic form

ln 2 = 5𝑟
Divide both sides by 5.
𝑟 = 0.1386 Use a calculator.
Your investment will double in 5 years at the approximate rate of 13.86%.

D. Applications
Learning Task 6.2.3: Think-Pair-Solve
Instructions:
1.Ely wants to double his savings in the shortest period possible. He is thinking in which of
the four banks offering him different interest rates and schedule should he invest his
money. Bank A offers a 3% interest compounded semi-annually while Bank B offers
interest rates of 2.8% compounded quarterly. Banks C and D offer 6% and 7% interest
rates, respectively compounded annually.
a. Which bank offers the best to satisfy his desire? Why?
b. Based on your answer in item 1, how many years will Ely’s money double?
c. How much interest will Ely gain from the bank which offers the best after 5 years?
2.How long will it take bacteria to double its population size if the growth rate is 5% every
hour?
135
Prepared by: Rennil S. Bornia MST-Math
Self- Instructional Module on College and Advance Algebra & Advanced Mathematics

3.How long does it take to earn $1000 in interest on a deposit of $6000 at 8% compounded
continuously?

IV. Assessment
Learning Task 6.2.4: Evaluation Quiz

Write each exponential equation in logarithmic form.

1. 52 = 25 3. 𝑎2 + 2 = 7
4. (𝑎2)2 = 𝑎4 6. 3𝑥 = 5

Write each logarithmic equation in


exponential form.
7. log2 32 = 5 8. log7 49 = 2 9. log10 0.01 = −2
10. log10 1 = 0 11. 3log8 2 = 1 12. ln 2 = 𝑥

Evaluate each expression without using a


calculator.
13. log9 81 15. log3 √27

17. log6 67
19. 3log3 5

Find the domain and range of each


function.
22. 𝑓(𝑥) = log2(𝑥 + 1)

26. 𝑓(𝑥) = log(𝑥 − 2) + log(2𝑥 − 1)

Match each logarithmic function with one of the graphs labeled a–f.
28. 𝑓(𝑥) = log𝑥 29. 𝑓(𝑥) = −log|𝑥| 30. 𝑓(𝑥) = −log(−𝑥)
31. 𝑓(𝑥) = log(𝑥 − 1) 32. 𝑦 = (log𝑥) − 1 33. 𝑓(𝑥) = log(−1 − 𝑥)

136
Prepared by: Rennil S. Bornia MST-Math
Self- Instructional Module on College and Advance Algebra & Advanced Mathematics

V. Enhancement
Learning Task 6.2.5:

1. Watch the YouTube video on Transformations of Logarithmic Functions with this link:
https://www.youtube.com/watch?v=bOrVW1wciqc
2. You may use a graphing calculator to graph each function and its transformations in the
same viewing window.
3. Then, describe an order for the sequence of transformations on the graph of 𝑦 = log𝑥 to
produce the graph of the given equation.
a.
b.𝑦 = −4 log(2 − 3𝑥) + 1
4. Based on the graphs and descriptions that you made, make a general procedures or
rules in graphing transformations of logarithmic functions.

VI. References
Ratti, J. M., & Marcus, S. (2014). College Algebra and Trigonometry (3rd Ed.). Pearson
Education Limited. Stewart, J., Redlin, L., & Watson, S. (2015). Algebra and trigonometry.
Cengage Learning.
https://www.youtube.com/watch?v=bOrVW1wciqc

137
Prepared by: Rennil S. Bornia MST-Math

You might also like